You are on page 1of 98

2017, 2018, 2019, 2021

MRCS Part A
April Recalls
APRIL 2017 RECALLS
1. What was that thumb abduction with hand on table answer:ABDUCTOR POLLICIS BREVIS
(supplied by median N).... Test: put hand on table and thumb pointing to the ceiling
IF UNABLE TO LIFT THE THUMB- IT IS FOR EXTENSION-HENCE TEST FOR ...both but EPL is more
2. what about injury in axilla and numbness in lat forarm:C6
3. source of blood during lumbar puncture ?lumber venous plx
4. type of femur fracture in the child who was hit from side/lateral side
?transverse/olique/greenstick/spiral
5. Woman with mets mx # same options-transverse
6. Tibial twisted-spiral# same
7. Child radial # - could be greenstick (if mild deformity/mild swelling)
8. Pt e prolonged catheter this frank hematuria and w loss :regional transional ca,diffuse
transitional ca/scc/intstisial cystitis
9. Cyst in breast aspirated cytology free what next? discharge
10. There was melanoma in situ removed? Discharge + self education for pt
11. One was audit of process
12. Second I wrote clinical audit but I cant remember the Q. 13. Pt e breast cancer and
metastatic disease come e hypotonia: hypercalcemia
14. After vericose vein surgery numbness dorsum of foot + unable to evert which nerve
damaged: CPN
15. Ttt of post op phelbitis: compression bandage+Non steroidal+elevation
16. Pt e antithrombin 3 Def and dvt (pt went for colectomy scenario): Warfarin life long
17. Rx for recurrent pulmonary edema: IVC filter
18. Surface anatomy of IJV ? From lobe of the ear to sternal edge of clavicle
If EJV: angle of mandible to mid one third of clavicle
19. Ulnar nerve , artery relation upper or lower forarm: ulnar to ulnar artery
20. There was this question about GCS. Patient localizes pain , open eye to pain and sounds
whats the GCS ? GCS 9

1
21. Peronial hematoma and lower abd hematoma? rupture
bladder,bulbar,membranous.prostatic
urethra ing.
Membraneous-blood on meatus, no urine, high riding prostate
Bulbar- perineal hematoma ,perineal swelling ,no urine
Bladder- urinary retention, abd pain, # pelvis displaced anteriorly, protinism
22. with leg flexion at 30 degree and moves lateral after trauma? Medial colateral injury
Valgus test: force inward to test medial colateral ligament (knee will not go inside if its intact)
Varus test: force outward to test LCL (if injury, knee joint will go outside)
23. Then a man hit by car when viewed from above left tibia lie posterior? pcl
24. Sciatic nerve injury what other movement not possible: extention of hip ?
Main flexor of hip: psoas major muscle
25. 6 yrs Child e sudden hip pain: perthes (features: xray normal, limping)
26. operating parathyroid and find nodule in 1.5cm thyroid what to do i chose
hemithyroidectomy( icant remmber the Qs and answer!)
27. Hemithyroidectomy for papillary 0.5cm nodule
28. Total thyroidectomy or medullary CA
29. Anexity adrenaline released from endings or gland ??gland adrenaline, from medulla
GFR:?/cortisol, androgen/sex hormone
30. Inj to the dorsal root ganglia???sensoey,motor,sympathetic ,parasympathetic,all of above
31. The one with cervical subcutaneous emphysema. Esophageal rupture :Ix of choice water
soluble contrast (gastrograffin):contrast study,CXR
*ANY PERFORATION: DO NOT DO BARIUM!
32. Ejection systolic murmur with pulmonary edema ; aortic calcification stenosis
33. Ascending or descending limb Ascending think loop of henle for furasmide
34. Then there was this methylsomething thiazide where did that act : dct
35. pt had pancreatitis thin hyperpara .. inv what next ?? Thyroid ?Pituitary? Adrenals
36. Factor which doesnt cause colon ca. Chrons, alcohol,smoking,diverticulitis

2
37. Was it gout or OA? Golfer with first metatarsophalangeal joint xray showed decreased
space
and sub chondral cyst: Rh arth, OA, Gout
XRAY sign for OA: subchondral cyst, narrowing of joint space, osteophyte, subchondral sclerosis
38. Man used opoid then constipation; painful fresh bleeding-fissure
39. Then was about prostate lymphatics internal iliac- internal illiac LN
40. Testis lymphatics: para aortic
41. Scrotum: transverse inguinal
42. Pt e vien graft 16 month ago then stenosis pathophysiology: Fibrosis, thrombosis or intimal
hyperplasia
43. Nerve to upper sctotum and penis ?? ilioinginal N
Ant scrotum - ilio inguinal
Periphery scrotum - genito femoral
Post scrotum- pudendal
44. Which artery anurysn compressed renal vein:SMA
45. ant relation to rt adrenal gland:IVC
Front: ?
Behind: diaphragm
Left: stomach, spleen
46. Portal vein drains To splenic Then splenic and SMV unite... IVC
47. Pt e head injury on IPPV,at the 5 day techypnea and hypoxia?
What to do? CPAP? CXR? Tracheotomy? swangang cathI?
48. Other q pt start weaning in icu? Same options above, SIMV (If no SIMV, choose
intermittent)/CPAP /tracheotomy
49. Patient underwent operation for IBD complicated by infection..now developed diarrhea...i
pseudomembranous colitis (c.difficile)
Clostridium difficile: transmission by feco-oral, given abx-->normal flora will be killed then it will

3
release enterotoxin A&B hence formation of crypts (mucupurulent material)
50. Esophageal perfiration type of feeding :feeding jejunostomy then go for TPN
51. Staged ediphagectomy:feeding jejunostomy
52. Perforated appendix organism? e.coli? Bacteroids?
53. Bilious vomting ,scaphoid abdomen And dudenal atresia
54. 9 month What was in child with rt upper quadrant mass empty rectum and vomiting?
Pyloric
stenosis?
RUQ mass, non bilious vomit: PYLORIC
55. There was a child with family history of cystic fibrosis, meconium ileus?
56. And then there was child vomiting with Distended abdomen And meconium plug was
renoved
after 3 days by suppository- hirschprung (early presentation of hsp)
57. Test for Addison des? short synacthen
Synacthen (Tetracosactrin) is a synthetic analogue, comprising amino acids 1-24 of the 39
amino
acid peptide Adenocorticotrophic Hormone (ACTH). This sequence retains the full biological
activity of intact ACTH. Synacthen stimulates the normal adrenal cortex to secrete cortisol,
which
can then be measured in serum. 58. Spleen rupture without trauma- EBV
59. study comparing fluid requirements? after burn in male n females.most patient had small
values and some larger values...what test to apply? Mann whittney (small value) /paired
/unpaired t test /chi sequare?
60. Case pt have 5 unite blood then bleeding profile: decr pltls ,normal ptt, incr INR, inc
fibroplastin norm fibrinogen products: DIC
61. intrinsic pathway? factor 9 (other intinsric factor: 8,9,11,12)
62. (Ascending Lymphangitis) creeping redness around vessels caused after thorne/gardening?
Strep pyogenes or clostridium perfringenes, staph saprophyticus
63. Root of transmesion of mrsa in some surgical ward 5 pt infected? Inhalation, contact(I
choose contact?) - skin to skin contact

4
64. Renal mass on USS do CT scan (without contrast),MRI,IVU
65. 15x9 cm lesion deep fascia,what to do: excision bx,inscion bx ,core bx (coz it is deep)
66. That liver accumulation prussian blue:iron (hemisiderosin)
67. Pt e history of FAP and colectomy done 30 yr ago now he came e jundice pale stool and
vomiting: deoudenal Ca *if Gastric ca in lynch
68. Melanoma suspected 1cm; Excise with 2cm margin?
If Pigmented mole suspected:excisional biopsy
If Pigmented mole with intermittent bleed and enlarged:excisional biopsy 2mm
If crusted, suggested melanoma: wide local
69. Patient 6 days post-op burn surgery with hiccups n abdominal distension...soft abdomen..
Acute gastric dilatation
70. Forehead mass pt itch cause bleeding follwed by incr in size during 1 yr:
bcc,scc,fibroadenoma
BCC features: Pearly, roll edges, erythamtous area, take years to growth
71. Which is the first immunologic cell in rosethorn and infl: Neutrophil
72. what does warming reduce levels of? TSH,ACTH,oxitocine,vasoprisin
73. Mediastinal mass with keratin and flat cells :seq cell ca
SCC features: keratin formation
74. Hyponatraemia in man post op with glucose fluids? Was it stress response or excess lack of
sodium containing fluids, extra fluid Na administration
75. Heat loss intra op by convection/ radiation
COMMON HEAT LOSS INTRA OP: Radiation (60%)--> conduction--> evaporation--> convection
76. Lung ca brain mets given: dexamethasone, paracetol, morphine
If mets to bone: radiotherapy
77. Marathon runner sudden death; SAH (berry aneurysm)
78. Nocturia ,thirsty - FBG:5 glucose then post prandial 2 hr was 7 – normoglycemia,impaired
glucose tolerece test ,DM,D.insipidus
-Fasting >7, post prandial >11 (to dx with dm)

5
79. What the hell do we give to pregnant lady with megaloblastic macrocytic: B12 first,folate
80. What about gastric acid secretion reduced by: Prostaglandin E2,ADP, -Gastric acid increased
lead to ulcer
81. What drug is a precursor to noradrenaline? dopamine
82. Duct papilloma proplimatuc : duct excision
Features: bleeding, old age
83. Post mastoidectomy loss of taste:chora tympany (2/3
rd anterior, taste, branch of facial nerve)
84. Follicular with bony mets-with forehead lump
85. Papillary with cervical lymph nodes
86. Then facial nerve - second arch
-First arch- mandibular arch/temporal
87. Pleuroperitoneal memberane for diaphramatic hernia
88. Surgeon clamped the lesser omentum (foramen of winslow) structures which is at direct risk
of injury:ivc, Common bile duct,hepatic artery
Post: portal vein
Ant to the left: common bile duct
89. J waves in hypothermia
90. Exostrophy bladder. Dorsal surface opening of meatus-epispedias (opening at the dorsal) -
Hypospadia (opening at ventral)
91. Nerves at risk Of injury surgical approaches:
92. medial ankle : Saphenous N
93. distal femur: tibial N
94. During hip arthroplasty surgeon note Artery run in superior border of pectenius ms ? Sup
gluteal
artery (given supply of neck of femur)
95. clamped umbiblical cord ,which will not carry oxigintated blood to the hrt? Ductus
venosis,hepatic artery,umbilical artery

6
96. Pulm stenosis for tof (VSD, PUL STENOSIS, OOA, ?)
97. Burn question with oedema, hypoalbumin
98. For onion peel: Ewing sacoma
99. Aortic dissection.. ascending or descending
100. thoracoacromial for deltopectoral groove:
medial-thoracoacromial artery
lateral-cephalic vein
*approach to axilla
101. Common peroneal for dorsum of foot loss and dorsiflexion loss? CPN
102. Smoking and microscopic was transitional cell ca
103. And egyptian farmer heamaturia was squamous cell ca
104. And the lft with raised amylase and ggt -alcoholic panceatitis
105. one was conus medullaris compression ...L1/L2 (in adult) L4 (in pediatric)
106. There was a uterus ligament question/Transverse perineal muscles
107. Pfenen – arcuate,rectus ms rt,liba alba,suprf.exter.rectus appon, ant rectus sheath
108. Median inscion:Linea alba
109. Inguinal:Ext obliques
110. Dartos in subcutaneous-from scarpa’s
111. Cremastic muscle - int oblique
112. Osteoporosis question NormoCa
113. Psycholosis,increase ca,tsh low then stop medication return to normal( milk alkali
synd,vit d toxicity,hyperthyroidism)
114. Keloid scar: intralitional steroid
115. Median and brachial; Lat ant medial (at the end: median will be medial)
116. X-ray failed to show scaphoid fracture...what next? Mri or ct scan? Or bone scan?
117. Parathyroid adenoma - 99-technetium sestimibi scan
If intra op: to identify gland is by methyln blue

7
118. extraintenstinal features of crohns except: polyarteritis
EXTRAINTESTINAL FEATURES OF CROHNS: Polyarteritis nodosum, pyoderma gangeronosum,
iritis, scleritis, asc cholangitis
119. There was a question about low potassium, high creatinine, scarred small kidneys? Atn,
polycystic, chronic pyelonephritis,etc
120. There was rcc for hg 18.3
Triad of RCC: hematuria, pain, mass+polycythemia
121. Cause of AAA?? Htn , artheroma , marfan,arthrosclerosis
Most common cause/risk factor of rupture: hypertension
Most common cause of acquired AA: artheroma,artherosclerosis
122. Efferent arteiole vasoconsrricted by Angiotension II
Renin from juxta-->convert angiotensinogen-->ang I (by ACE in lung/liver)-->ang II--
>vasoconstriction
123. Pcwp 20= pul oedema
124. Pcwp 11= ARDS
125. High riding prostate.... Membranous
126. Pineal hematoma+ excessive collection..Bulbar
127. Cushing... Hypok
128. Metabolic acidosis... Hyperk
129. Wide qrs... Hyperk
130. The aortic valve replacement drop in hr and bp?heart block, temponade
131. Gall bladder stone?bile pigment
132. Sickle cell disease +bone pain+pneumonia agent: streptococcus pneumonia
133. Pneumonia in ventilated... Actinobacter (choose this if not in option, choose
Psedomonas (drugs:tinam IMIPENEM) .sterp pn
134. Damage to 3.5 finger sensation :median N
135. Guys there was a question about achilles tendon ruptute what was the test? Simmons
Thompson squeeze test

8
136. What do they clean plastic syringes with? Radiation
137. What abt collection when patient lying supine: Hepatorenal pouch of morisons, paracolic
gutter
138. Acute abd pain ,absolute constiopation,55yrs old abd x ray shows Small n large bowel
Air fluid level: colorectal Ca,adhisions,sigmoid volvolus
139. Tricuspid closes - isovolumetric contraction
140. Pathophysiology ARDS: Reduced diffusion of air (lung will be stiff)
141. Tachy + cardia chest pain: Decreased diastolic interval
142. Tricuspid closes - isovolumetric contraction
143. Vital capacity: full expiration and full inspiration
TLC: vital capacity + residual volume
144. eggs at anal verge: Mebendazole or metronida?
145. Cleft palate posterior to incisive foramen due to? Malunion of maxillary n nasal
prominences (for cleft lip) i choose palatine shelved
146. There was a q about multiple lytic lessions in pelvis? It was multiple myeloma
Urine: bence jones, xray: lytic lesion
147. *eosinophil q, type 1 hypersensitive
148. *carcinoid,octreotide
149. Hassal corpuscles: yes thymoma
150. Gout-needle shaped negative brifringent under scope
151. Breslow depth less than 1mm good prognosis? THE LESSER THE BETTER
152. There melonoma in situ excised with 1 cm margin . Education of self exam
153. chemoradiotheapy...only that sounded reasonable for metastatic breast Ca.…
Inflammatory breast ca-chemoradiotherapy (if pt give abx and not responding at all)
154. Vesicoureteric stone at? tansverse process L5-ischial spine
155. best prognosis in breast ca:Er+ve, HER-ve
156. Something to do with infection spreading to brain/inner ear was it sigmoid sinus/Inner

9
ear or what?sigmoid sinus
Infection from ear to brain: Mastoid air, tigment timpanid by roof of middle ear
Choroid plexus, opthalimic v, emessary v (dangerous area)
157. Brown sequard
158. little's area of nose artery? Sphenopalatine
159. anterior wall of the heart ? Mostly made of rv n ra
160. rt Coronary a arise .arise from post surface of aorta.arise from ant surface .lei in post
Interventricular groove .arise inf to aorta from sinus(Rt coronary ... ant surface-Lt coronary ...
post surface)
161. Depolarizing neuromuscular blocker... suxamethonium
162. Cervical sinus from what branchial cleft?2nd
163. Cleft palate post to inscive foramin what developmental anomaly?palatine shelves
164. Insertion of ureter in bladder:post
165. Thoraco acromial a injured in deltopectoral groove
166. Second q ant to which part of vertebra ureter insertion
167. Q asking about poas major(T12-L5)
168. Realtion of ulnar artery ana ulnar nerve in wrist(ulnar to ulnar)
169. Ant surface of heart formed by what
170. Nerve lesion in hernia operation cause parathesia at top of scrotum and base of penis
171. Ant relation to rt adrenal gland
172. Ewing sarcoma onion
173. The 4 cases about mechanism of injury
174. Green stick fracture
175. 2 cases wara b3d bilateral lung opacity in the 2 cases but pulmonary wedge pr is high in
one of them I choose for one pulmonary embolism ARDS
176. Artery in epistaxis
177. Q about osteomalacia

10
178. Q about multiple myeloma
179. After the nurse cut the umbilical cord which of the following will not contain oxygenated
blood any more Ductus arteriosus- ductus venosus-portal v -umblical a-hepatic ar
180. Invetsigation for boerhaave syndrome
181. This first senario was AAA with low PAWP one post op
182. The second was following pancreatectomy a 3days later low PAWP
183. MEN1 pituitary
184. Medullary ca Tx
185. Follicular Ca Tx
186. Addison > hyperkalemia
187. TB medication adrenal crisis
188. Recurrent PE vena cava filter
189. Thrombin 3 deficiency
190. Anasthesisa of pubic , scrotum following herniorrhaphy
191. Impaired OGTT
192. cAMP decrease gastric acid production
193. Noradrenaline septic shock
194. Three audit q came in first paper
195. Then about 6 cases asking about electrolytes
196. 2 Q parkland formula
197. Preganglionic neurotransmitter - i marked ach
198. Resting potential of skilital ms maintained by(Na,K,Ca,Cl)
199. 60 yr old man e itchy and painless yellowish discularation
200. The question was asking about tuberclin test type of hypersensitivity as I remember:
Type 4,T cell mediated
201. Invest oesophageal rupture???baruim salaow
202. Deud ulcer:GD art.

11
203. Dumping
204. Hip extension due to semitendinosus
205. There was Q about ulcer deep to it swelling contain keratinized skin calcium and bone
What is the pathology:scc
206. Premature baby with inguinal tender swelling.. direct hernia, indirect, femoral, testis
torsion, inguinal lymphadenitis?
207. St segment depression in inferior leads:inf MI
208. A severe COPD patient receiving high flow oxygen became drowsy...?acute resp failure
209. Predominant cells in granumola/tb...macrophages
210. Wide qrs ....hyperkalemia
211. High jejunal fistula? Electrolyte imbalance
212. A sever copd with peritonitis,He was metabolic acidosis due to peritonitis
213. Organism that causes diverticular abscess in adults.. 214. patient presented with knee pain
and operted in his left knee after 24 hour he develop
chest pain and hemoptysis shortness of breathing what underline heamtalogical predispose to
his s presentation??? Anemia,Thrombocytosis,Thrombocytopenia,NeutrophilIi
215. Extended match question;Hyperkalemia,Hypokalemia,Hypercalcemia,Hypocalcemia:
216. 1_wide qrs on ECG
217. 2_absent p wave long QT interval
218. 3_high jujenial fistula
219. 4_cushing syndrome
220. 5_corrected metabolic acidosis
221. Tension pn.thorax
222. Sings of elevated ICP No history of any truma and morning headache nausia moring
bilateral papiloedema :frontal glioma ,chronic subdural bleeding,subarachnoid hg
223. Most common cause of hydrocephalus:f.monro ,f.magende ,luska,aquidecut
224. which sensation lost on the thumb and lateral side of distal forearm........answer was

12
post interosseus or radial??
225. Penut inhalation :rt lowe pole
226. +ve pleural pr in :valsalva
227. Medial border to femoral canal:lacunar
228. Pt in sheet fawzia:metabolic acidosis due to pertonitis
229. Not happen in crhons :poly artritis
230. Swelling of whole arm after exercising how to investigate :selective axillar ar
angiogram,duplex,uls?
231. Atrial deporalzation: p wave
232. Spinal acessory post triangle neck
233. Pulm arts come from 6
th arch
234. Doesiflexion L5
235. Qs about Pulm valve lt 2nd intercostal space?
236. Pips decrease preload
237. S3=perianal region
238. There is q about chrons disease then have watery diarrhea:hypokalemia

13
APRIL MRCS A 2018
1. Posterolateral hernia in child…..reason?
a. Failure of closure of right pleuroperitoneal membrane
b. Failure of closure of left pleuroperitoneal membrane
c. Failure of closure of septum transversum
d. Failure of closure of contribuiton fromnthe thoracic cavity wall

2. A statistician wanted to compare some parameter which was not distributed normally (the
q mentioned few patients had extremes of values) between two groups
a. Chi square
b. Mann whitney U
c. Paired T test
d. Unpaired T test
e. Kruksal wallis

3. 2 cm mass in lung in a smoker, another 1 cm mass in liver suspected to be hemangioma. No


systemic signs, best investigation NEXT?
a. MRI liver
b. MRI whole body
c. CT chest
d. FDG PET CT
e. USG

4. Loss of weight, cough, tiredness, Multiple lymphadenopathy, Lateral aspect of tongue


hairlike lesions
a. Lichen planus
b. Candidiasis
c. AIDS

5. Nerve injury, deltoid affection, which other muscle will be affecting ..teres minor
6. Fat overweight child, pain hip, external rotation on attempting to flex … Slipped femoral
epiphyses
7. 6yo child pain hip in all range of motions, systematically well, no fever Perthes
8. Trauma to knee, swelling, excess lateral anguity on stress testing, injury to… MCL
9. Injury to knee and swelling, on 90 degrees tibia is more posterior than other side… PCL
injury

10. 3 patients in surgical ward with C. Difficile, mode of transmission… (MRSA CONTACT)
a. Endogenous
b. feco-oral
c. nosocomial
d. respiratory

14
11. Test for rupture tendon achilles… Simmonds squeeze test
12. Sinus tarsi between bones?
a. between talus and calcaneus
b. Talus and navi
c. Calc and navi

13. Lift thumb with hand 90 degrees on table (retropulsion test) … EPL
14. Mobile extensive femoral thrombus management initial management
a. Heparin
b. Venous exploration, c. Aspirin, d. Embolectomy

15. Which hormone decreases in warm condition…


a. TSH, b. ACTH, c. Vasopressin

16. Cleft palate embryology defect in fusion of what structures? … palatal processes
17. Result of failure to develop caudal portion of metanephros………….Kidney
18. Testicular cancer with normal AFP and HCG
a. Seminoma
b. Teratoma

19. Relation of median nerve with respect to brachial artery in proximal arm… lat to ant to
medial
20. Relation of ulnar nerve to ulnar artery… ulnar to the ulnar artery (ulnar N is medial than
A)
21. Lesion in Dorsal root ganglion in neck, what type of loss?
a. Sensory
b. Motor
c. Sympathetic
d. Parasympathetic
e. All of the above

22. Atrophy thenar eminence… median nerve


23. Dermatome of the little finger… C8
24. Injury to esophagus during upper GI scopy ?
a. TPN
b. Nasoenteral feed
c. PEG
d. Elemental diet

25. Ix for rupture esophagus after endoscopy…


a. water soluble gastrograffin
b. Barium (can lead to mediastinitis)
c. upper GI scopy

15
d. Endo USG
e. Chest xray

26. Crohns …operated… resection of distal ileum and many stricturoplasties?...what


nutrition??
a. Elemental diet
b. High nutrition oral or enteral diet
c. TPN

27. Reciprocal of absolute risk reduction …NNT


28. Cushing reflex/triad… ↑BP ↓HR ↓RR (irregular breathing)
29. Athlete sudden death after marathon
a. SAH (berry aneurysm)
b. IVH
c. EDH
d. SDH
e. Raised ICP

30. 6yo, kid with groin swelling, observed when the kid is playing around, which
spontaneously reduces on lying
a. patent processus vaginalis
b. direct hernia
c. femoral hernia
d. indirect hernia
e. tortion testes

31. Child with pain in the right scrotum with redness and irreducible groin mass
a. direct hernia
b. femoral hernia
c. indirect hernia
d. tortion testes

32. Intrinsic pathway…factor Xa


33. Old woman displaced # distal radius, reduced and put in slab/POP for 6 weeks,
complaining of pain and limited ROM in both supination and pronation after 3 months…
a. CRPS
b. Malunion
c. Nonunion

34. Cervical lymph node, FNAC squamous cell ca, upper tract scopy was found to be normal.
What investigation next ?
a. CT Neck
b. MRI neck
c. FDG PET CT

16
d. MRI whole body

35. Diarrhea, pruritus ani, cellotape examination shows egg in stools…


a. Mebendazole
b. Metronidazole
36. Some q on clinical scenario of hypercholesterolaemia….
a. Hypothyroidism decreases cholesterol
b. Hypothyroidism increases cholesterol

37. Advantage of carbohydrate rich drink in enhanced recovery program?


a. decrease post-operative insulin resistance and negative nitrogen balance (ERAS)
b. better abdominal muscle function
c. better respiratory muscle function
d. shorter hospital stay

38. What’s in direct contact with T5 body…


a. azygos,
b. left atrium,
c. aortic arch OR its junction with ascending aorta

39. Shoulder pain between 60-120 degrees… supraspinatus tendinitis


40. Gun shot to on right side below costal margin, injury to...
a. pylorus,
b. gallbladder (fundus of GB)

41. Some trauma, led to UMN lesion of leg muscles. Where can be the lesion?
a. L1-2,
b. T12-L1 (below L1 is LMNL)
c. L2-L3
d. L3-L4

42. Chinese man with LN enlargement and headache and deafness nasopharyngeal carcinoma
(by EBV)
43. Spastic unilateral leg paraplegia which artery
a. ACA
b. MCA
c. PCA
d. Ant choroidal art

44. Impotence and vascular insufficiency, clue to Leriche syndrome occlusion to both common
iliacs
45. Femoral canal lateral border… femoral vein
46. Diagnosis of DVT by… compression US (if gold standard: VENO)
47. Patient with Barret’s esophagus… adenocarcinoma

17
48. Golfer with 1st Carpometacarpal joint pain, XR showed narrowed joint space and
subchondral cysts… OA
49. Gangrene of the anterior abdominal wall…Cl. perfringens
50. Post-partum lady dies after 3 days causative organism… (???)
51. Osteomyelitis causative organism… Staph aureus
52. Brown sequard syndrome... ipsi motor weakness and proprioception + contra pain and
temp
53. Hypercalcemia in a metastatic scenario, Ca 3.7 mmol/L, ECG abnormalities, “Initial”
treatment…
a. IV 0.9% sodium chloride
b. Steroids
c. Pamidronate
d. Cinacalcet

54. Hypothermia ECG change… J wave


55. 1cm melanoma over the back, best management…
a. excision biopsy with 2mm margin,
b. excision biopsy with 2cm margin
c. core biopsy

56. 23-25 yr old female, single mobile breast mass, most appropriate investigation
a. USG
b. Mammo
57. Site of action of fursemide…thick ascending loop of Henle
58. Site of action of thiazide… DCT
59. Male gynecomastia which drug… spironolactone
60. Posterior duodenal ulcer collapse, which artery… gastroduodenal artery
61. Site of Dartos muscle subcutaneous
62. Waddling gait, injury to… superior gluteal nerve (gluteus medius muscle)
63. Bypass with graft, later is blocked… neo intimal hyperplasia
64. COPD patient… ↑CO2 ↑HCO3 (respiratory acidosis)
65. Anterior relation to right adrenal…IVC
66. Good prognosis for melanoma…
a. Breslow thickness of 0.6
b. Clarks level 5
c. Lymphatic invasion
d. Satellite nodules
e. Ulceration

67. 62% burn patient presents with lower leg edema after 3 days…(pain was not mentioned in
the q)
a. Hypoalbuminemia
b. DVT
e.coli and bacterioids

18
68. Picture of x-ray with arrow I think was pointing at a structure at C3 level
a. hyoid bone
b. cricoid
c. thyroid
d. epiglottis

69. Male with pain in testes and urethral discharge…


a. Gonorrhea
b. Chlamydia

70. Blood supply to transverse colon… middle colic artery


71. CPP calculation…BP was 125/80, ICP was 20….75
[calculation for CCP=0.333 (systolic-dystolic)+dystolic - icp]
72. Submandibular duct excision injury to… lingual nerve
73. Post parotidectomy patient received radiation treatment, followed by presentation with
dryness of eyes, which ganglion involved…
a. Pterygopalatine
b. Otic
c. Ciliary
d. submandibular

74. Injury to head, fixed dilated pupil, cause…


a. unopposed sympathetic fibers
b. unopposed parasym
c. overaction of parasym

75. Osteoporotic crush to vertebra, serum calcium is likely to be…


a. Hypercalcaemia
b. Hypocalcaemia
c. Normocalcemia

76. Some clinical scenario, hypercalcaemia, given steroids, resolved………sarcoidosis


77. HPE in rheumatoid arthritis?
a. Necrobiotic granuloma
b. Foreign body giant cell with lymphocytes
c. Macrophages
d. Eosinophilic granuloma

78. Fall in blood BP, 1st reaction…


a. sympathetic system
b. RAAS

79. 11yo child with special need, going for laparoscopy, type of consent… (consent parents)
80. Ligation of splenic hilum, avoid injury “during the surgery” to what…

19
a. tail of pancreas
b. greater curvature

81. Ureter stone radiating to groin, which nerve root… (T11-L2)


82. What structure is in front of uncinate process… SMA
83. What forms the anterior surface of the heart
a. Mostly Rt atrium and ventricle
b. Mostly left ventricle

84. Pigmentation,,,, clinical picture like addisons, electrolytes Na 128, K 5.8 or 6.8, which
hormone to check (ACTH)
85. Some scenario with single site metastatic bone pain treatment…
a. radiotherapy
b. NSAIDs
c. PCM
d. steroid
(follow analgesic ladder first before choosing radiotherapy)

86. Ovarian mass and inner thigh numbness, which nerve… obturator
87. Bilateral cervical lymph node spread, which type of cancer…tongue
88. Skin lesion, pearly… BCC
89. Skin lesion, central keratinous something…keratoacanthoma
90. UK NICE guidelines on people who received transfusion
a. can’t donate in future until viral markers are negative
b. Can donate within 6 weeks
c. Can donate within 12 weeks
(correct ans: can never donate)

91. Anterior compartment syndrome affecting… 1st dorsal web space


92. Pathology of berry aneurysm in 45yo…
a. Atherosclerosis
b. Cystic medial necrosis

93. site of lumbar puncture…


a. at the level of iliac crest
b. at the level of transtubercular plane
c. at the level of posterior superior iliac spine
d. at the level of posterior inferior iliac spine
e. at the subcostal level

94. ulcer at the tip of penis


a. squamous cell ca
b. adeno
c. transitional cell ca

20
95. Ix for pheochromocytoma… urinary VMA (if serum: metanephrine)
96. post-op day 6 , operated after trauma, with continued bleeding PT 18 secs, APTT 60 secs,
Fibrinogen 0.1 … (DIC)
97. a hypertensive taking Ramipril. Q was , constriction of efferent arteriole by:
a. angiotensin 1
b. renin
c. angiotensin 2
d. aldosterone
e. ANP

98. some guy with a knee swelling and sudden pain in legs after some 7 days which and
scenario of Moses sign although the name wasn’t there, with disappearance of knee swelling
…DVT???
99. caecal cancer, resected, patho gave diagnosis T4, what do u mean by that?
a. Involving surrounding abdominal musculature
b. Multiple lesions in the resected specimen
c. Muscle invasion
d. Lymphatic or vascular invasion.

100. post op patient with known Graves, P 120, BP 160, treatment after salvaging then acute
phase
a. betablocker + iodide
b. betablocker + thioamides
(start with (B) in the acute phase after resolve continue ttt with (A))

101. Best investigation of choice for parathyroid adenoma


a. Sestamibi scan
b. USG
c. CT
d. MRI

102. Can’t remember, most likely risk factor for HCC… (alcohol)
103. Some 40s male, nausea, vomiting, blurring of vision and headache, particularly in the
morning with no focal neuro signs
a. Raised ICT
b. Frontal glioma
c. EDH?
d. SDH?
e. SAH?

104. Clinical scenario of MM, HPE what to expect


a. Plasma cells
b. Macrophages

21
c. Lymphocytes
d. Eosinophils

105. Asian middle aged man, found to have cavitatory lesion at the apex of left lung, HPE
expected … gohn complex formation (TB)
106. Associated with epispadias……(extrophy of bladder)
107. Hypospadias…developmental anomaly of urogenital fold
108. Can’t remember, some question about diffuse lung infiltrates or ARDS, cause of clinical
picture?
109. Complete expiration from complete inspiration….lung volume?
a. VC
b. TLC
c. FRC
d. ERV

110. 30 yr old man, presented with Mediastinal mass. FNAC shows glandular pattern, absence
of neuroendocrine markers, ill defined clumps of cells.
a. Adenocarcinoma
b. Thymoma

111. Some trauma or surgery on condyles of femur or tibia. Followed by loss of pulsations.
Injury? .... (popliteal)
112. Patient with liver laceration, during ot clamp free margin of lesser omentum. Possible
damage to?
a. CBD

113. Post cardiac transplant… what increases Cardiac output?


a. Increased atrial filling
b. Increased intrathoracic pressure
c. Increased intrapericardial pressure
d. Some option acting via autonomic nervous system

114. Injury to axilla, numbness lateral forearm, which muscle will be affected as well?
a. Brachialis- supplied by musculocutaneous N (Lateral cord)
b. Brachioradialis
c. Flexor carpi radialis
d. Pronator teres

115. Can’t remember, question about heart valve component options had
a. Dense fibrous tissue
b. hyaline cartilage
c. smooth muscle

116. Ureter relations,

22
a. Gonadal veins Cross anteriorly
b. Common iliac divide anteriorly

117. pain on walking down hill but not uphill… lumbar canal stenosis
*if young girl, difficult walking stair-chondromalacia
118. patient post burn, with hematemeses… curling ulcer, acute gastric dilatation
119. multiple bone pains, on exam, ill defined hard prostrate , PSA 300. Management?
a. TURP,
b. radical prostatectomy,
c. incision of prostrate
d. hormonal therapy
e. active surveillance
(if hormonal therapy not present choose radiotherapy)

120. long standing venous ulcer, swabbed , found to have MRSA, no signs of infection given in
the q
a. oral lincopeptides( Vanco or Teico)
b. other beta lactams
c. beta lactams
d. beta lactams
e. no antibiotics

121. effect of smoking on body-lung, bladder, breast, tongue


122. trauma patient, “properly resuscitated”, when log rolling , suddenly became hypoxic or
decompensated
a. displacement of tracheal tube
b. spinal injury

123. history of carrying heavy weight with back pain….facet arthropathy


124. tooth extraction, bleeding stopped then, started after some time.Factor VIII (factor 8)
125. mass in breast of 25 yr lady, mobile, non tender, what inv ?
a. USG
b. Mammo

126. Lady underwent mastectomy. Split skin graft taken from thigh. What type?
a. Allograft
b. Isograft
c. Autograft
d. Meshed graft
e. Xenograft

127. Post op patient of zygomatic repositioning, ear discharge, deafness …#petrous part
128. Pharyngeal pouch, between which muscles?... between Thyropharyngeus and
cricopharyngeus

23
129. 16 years old boy with gynaecomastia …..physiological
130. Renal mass noted in USG. Next investigation?
a. CT
b. MRI
c. FNAC
d. PET

131. Vessel originating above piriformis….sup gluteal A


132. Trendelenberg test +ve……. gluteus medius (supply gluteus medius m)
133. Post op patient deteriorating picture, P 60-70, BP 80/50, CVP 20
a. Heart block
b. Sepsis
c. Cardiac failure

134. Acute abdomen, amylase 110 [normal below 100], Ca 2.4 [2.1 – 2.8], LFT normal
a. Acute intermittent porphyria (can see in lead poisioning)
b. Perforation
c. Pancreatitis
d. Rest of the options I don’t remember , but nothing matching the q

135. Type 1 Diabetic patient on long term NSAID for osteoarthritis or some painful condition,
post op creat high, urea high
a. Reduced renal perfusion
b. Diabetic nephropathy
c. NSAID induced renal injury

136. Etiopathogenesis of AAA……..atherosclerosis


137. Patient known case of iliac or femoral atherosclerosis or aneurysm. Came with sudden
onset pale pulseless limb….embolism
138. Patient with known periph vasc ds, underwent fem-distal bypass 1 yr back, now came
with painful limb, DPA not palpable, but doppler showing monophasic flow until ends….DVT
139. Mass in breast, with eczema of nipple areola. Q was , apart from mammo what else will
u do?
a. Exfoliative cytology
b. FNAC
c. Core Biopsy
d. MRI
(Skin Biopsy)

140. Which of the following is true about Rt coronary artery?


a. Originates posteriorly from aorta (not sure but in discussion says A)
b. ………………anteriorly…………………
c. Originates from aortic sinus below the aortic valves
d. Present in posterior IV groove

24
141. Angle of loiu…junction of arch and descending
142. Another q Structure at/attached to T5..
a. Left atrium
b. Tracheal bifurcation
c. Bifurcation of pulmonary trunk
d. Bifurcation of right bronchus

143. Some surgery led to injury to thoracic duct, surgeon wants to ligate it from thoracic
approach. Where to find
a. Behind ascending aorta
b. Venecaval opening
c. Esophageal opening
d. Aortic hiatus

144. Post appendectomy, pus collection in…..


a. rectouterine pouch
b. vesicouterine ”
c. behind the rectum
d. in front of bladder

145. Scrotum… (superficial inguinal)


146. Ovary… (para aortic)
147. Below anal verge (superficial inguinal)
148. Middle aged man, testicular mass, HPE shows cells separated by fibrous septa containing
lymphocytes…
a. Classical seminoma
b. Spermatocytic seminoma

149. Surface marking of IJV


a. styloid process to medial end of clavicle
b. ear lobule to medial end of clavicle
c. mastoid process to medial end of clavicle
d. lesser cornu of hyoid to medial end of clavicle

150. myasthenia……antibodies to acetylcholine


151. NSAIDS causing ulcer…by what mechanism … Cyclooxygenase
152. Mechanism of peptic ulcer
a. Mucous cells
b. Chief cells
c. Oxyntic cells
d. Parietal cells

25
153. Gunshot which entered at the junction of right rectus and right costal cartilage and exits
exactly from the opposite. Structure most likely injured ?
a. GB (precisely fundus of GB)
b. Renal hilum
c. Pylorus

154. Some surgery on leg done under spinal with anaesthesia upto umbilicus…..T10
155. Patient with 9th 10th 11th and 12th cranial nerve lesion… medulla
156. Cause of metabolic alkalosis?
a. Nasogastric aspiration
b. Alcohol

157. Post op gustatory sweating….. parasym fibres developing into sweat glands (seen in
Frey’s)
158. Some scenario and inj ampicillin or amoxicillin given; soon presented with scenario of
anaphy rxn. Mediator?
a. Mast cells
b. Eosinophils
c. Lymphocytes
d. Macrophages
EMQ Bladder
159. Egyptian, hematuria… SCC (organism: Schistosoma hematobium)
160. Female long history of indwelling catheter and multiple recurrent infections, presents
hematuria…SCC
161. Male in dye industry…TCC

EMQ on Ear
162. Discharge from ear for a 10 years, with middle ear perf and facial nerve
palsy…cholesteatoma
163. … ()
164. … ()
165. Some kind of mass in neck? Don’t rememebr ……….Otitis media with effusion

Stupid EMQ on Jaundice


166. Man with pain, jaundice, fever….had similar episodes of pain abdo
previously….choledocholithiasis (cholangitis was not in options)
167. 30 yrs follow up patient of FAP operated for colon, now presented with Oseophageal
Junction… Carcinoma of duodenum

EMQ Fluid burn calculation in 24h


170. 36% burn in some 60 or 70 kg
(PARKLAND’S FORMULA)--> 4ml X BSA (%) X BW (kg)
4x36x70= 10080ml
10080ml/2= 5040ml

26
Hence-->

If FLUID MAINTENANCE:
100mls/kg for 10kg of weight
50mls/kg for 10kg of weigh
20mls/kg for remaining 10kg after
171. 19 yr old with 90 kg weight with burn of a leg and perineum

EMQ chest trauma


i. Chest drain
ii. Chest drain with suction
iii. CT
iv. Echo
v. Thoracotomy
172. Stab to right 4th ICS, normal CXR, tachy, rest normal-  CT
173. Stab to the left 5th ICS, tachy, rest normal- ECHOCARDIOGRAM
174. Stab to 5th ICS post axillary line, CXR showing pneumothorax with fluid level - CHEST
DRAIN

EMQ Chest pain in pregnancy


175. 36 weeks, 28 yr, c/o Chest tightness and dyspnea. On exam, cyanosis and one more
alarming feature. Dad died of AMI at 62 yrs…..AMI
176. 36 weeks, 28 yrs, father died of AMI at 42 yrs, chest pain, no other info.
Exceptionally(word used was some different) tall….. Aortic Dissection
177. 36 weeks, 28 yrs, Preg lady with pleuritic chest pain followed by dyspnoea, father died at
60…. PE

EMQ Child gastrointestinal disorders


178. 6 months, sausage shaped mass…intussusception
179. 6 weeks non bile stained vomit after feeds…pyloric stenosis
180. Tender distended abdomen, per rectal bleed in 6 weeks baby……midgut volvulus
181. Dark red stool with pallor- meckel’s

EMQ GCS
182. E2V2M2
183. E1V1M1

EMQ Ortho patterns of fracture


184. Twisting force… spiral
185. Child fracture with bowing happening after injury greenstick
186. Direct blow to femur from side… transverse
187. Fracture humerus with metastatic lesion, mechanism was fracture happened while she
was trying to aboard a bus and got pulled by the running bus  Oblique

27
EMQ Ortho nerve injury
188. Posterior approach to femur… Tibial nerve
189. Humerus posterior approach… radial
190. Medial approach to ankle… saphenous nerve

EMQ urethral injury


191. Pelvic fracture, unstable patient, DRE prostate unusual location… rupture membranous
urethra
192. Fall with legs wide open on handle bar bulbar urethra injury

EMQ plastic
i. Full thickness graft
ii. Partial thickness graft
iii. Pedicled flap
iv. Microvascular free flap
v. Advancement flap
vi. Wound excision and primary closure
vii. Wound excision and 2nd ry closure
viii.
193. Pit bull bite off a substantial portion of nose-WOUND EXCISION AND HEALING BY
SECONDARY CLOSURE
194. Cut wound scalp with dirt-WOUND EXCISION AND PRIMARY CLOSURE
195. Garden fork in toe, 1cm dusky wound-WOUND EXCISION AND SECONDARY CLOSURE

EMQ Thyroid
196. 1 cm MCT … total thyroidectomy
197. 0.5 cm nodule found in one lobe of thyroid while operating for another surgery probably
parathyroid…hemithyroidectomy
If papillary >1cm--> total thyroidectomy
If adenoma--> do hemithyroidectomy
17 years old, Pt young, fit going for tonsillectomy, what ix? Coagulation profile?--> choose
nothing of the above

EMQ Types of biopsy


198. Venous ulcer punch biopsy (if no punch in option--> choose incisional biopsy)
RUO mass 3cm: cope biopsy
Retroalveolar mass: core biopsy
Paget disease: punch biopsy (if no option for punch or incisional--> choose skin biopsy)

EMQ on Splenomegaly
199. Caucasian man, antibodies to EBV +... (Infectious Mononucleosis)
200. 16years old, Caucasian girl, mild jaundice+stone, spleenomegaly…spherocytosis
Calf pain +homan’s test +ve-->DVT

28
EMQ on Acid Base
K+ Na+ HCO3
A 2.6mmol/L 148mmol/L 36mmol/L
B 3.4mmol/L 156mmol/L 24mmol/L
C 3.2mmol/L 118mmol/L 22mmol/L
D 6.0mmol/L 120mmol/L 15mmol/L
E 2.8mmol/L 128mmol/L 14mmol/L
NORMAL 3.5-4.9 137-144 22-26
A 65 years old man is admitted with 2 week h/o vomitting. Ix confirm benign pyloric stenosis
ANSWER IS = K 2.6 NA 148 HCO3 36
A 40 years old woman is admitted to ER having collapsed. O/E abdomen soft, no sign of
peritonitis. Pigmentation of mucous membrane of the mouth is noted. Ix reveal hyptension
(80/60), sinus rhythm, a normal CXR and abdominal US. Her only previous history is an
unexplained anemia and thyroxine replacement therapy for an immune thyroiditis.
ANSWER IS = K 6.0 Na 129 HCO3 15
42 years old man undergoes small bowel resection for CD. 4 days postoperatively he develops a
high output intestinal fistula. Ix reveal the fistula to originate from a small bowel anastomosis
situated in the upper ileum. Fistula losses amount to 1800ml/24H
ANSWER IS= K 2.8 Na 128 HCO3 14

CLAUDICATION!!
If Calf: sup. femoral
If Thigh: external illiac
If Buttock: common illiac
If Buttock+impotence: leirche’s syndrome (both com iliac)- take time around 3 month

29
APRIL 2019 RECALLS
1. A 26-year-old man presents to the Emergency Department with extensive bleeding from
his arm after sustaining a glass injury. On examination there is a 7 cm transverse laceration
across the anterior aspect of his elbow. On exploring the cubital fossa, you would expect the
brachial artery to be:
A. anterior to the median nerve
B. lateral to the biceps tendon
C. lateral to the median nerve
D. medial to the median nerve
E. superficial to the bicipital aponeurosis

2. A 19-year-old woman presents to the Emergency Department profoundly hypovolemic


having fallen from a horse. A postero-anterior (PA) chest radiograph shows a fracture to the
medial third of the left clavicle. Which of the following vessels is most likely to be damaged?
A. Brachiocephalic trunk
B. Left axillary artery
C. Left common carotid artery
D. Left subclavian artery
E. Left vertebral artery

3. A patient can't dorsiflex his big toe, which nerve root is most likely to be involved?
A. T12
B. L3
C. L5
D. S1
E. S2

4. A Patient fell at his wrist resulting in loss of sensation over the palmar aspect of the lateral
3 and half fingers. What is the expected injury causing this?
A. Scaphoid fracture
B. Lunate dislocation
C. Distal radial fracture
D. Distal ulnar fracture
E. Hamate fracture

5. A 28-year-old man presents with pain in his left scrotum. A diagnosis of varicocele is made.
Which vessel is involved?
A. Gonadal vein
B. Inferior epigastric vein
C. Pudendal artery
D. Artery of the vas
E. None of the above

30
6. During recovery from varicose veins surgery, a 35 years old woman complains of weakness
of dorsiflexion of the ankle. Physical examination reveals absent sensation over the dorsum
of the foot. Which of the following nerves is most likely injured?
A. Common peroneal nerve
B. Deep peroneal nerve
C. Saphenous nerve
D. Superficial peroneal nerve
E. Sural nerve

7. A 70-year-old man undergoes transurethral resection of bladder tumour (TURBT). The


tumour lies over the lateral side wall of the bladder just above the opening of the right
ureteric orifice. Use of the diathermy suddenly causes the patient to 'kick' because of
contraction of the hip adductor's muscles. Which nerve has been stimulated?
A. Femoral nerve
B. Genitofemoral nerve
C. Lateral femoral cutaneous nerve
D. Obturator nerve
E. Sciatic nerve

8. A patient sustained a fracture tibia for which he was put in a cast for 5 weeks. He then
developed severe pain in his leg with pain on plantar flexion. Where would you expect
sensory loss?
A. Lateral aspect of the foot
B. Medial aspect of the leg
C. Dorsum of the foot except 1st web space
D. First web space
E. None of the above

9. What is the corresponding dermatome of the little finger?


A. C8
B. C7
C. C6
D. T1
E. C5

10. A patient became unable to dorsiflex his big toe. What is the corresponding spinal root?
A. S1
B. L4
C. L5
D. L3
E. S2

11. What is the relation of ulnar nerve to the ulnar artery at wrist?

31
A. Ulnar nerve is ulnar to ulnar artery
B. Ulnar nerve is radial to the ulnar artery
C. Ulnar artery is ulnar to the ulnar nerve
D. Ulnar nerve is lateral to the ulnar artery
E. None of the above

12. A 70-year-old woman from a nursing home is brought to the Emergency Department with
abdominal pain and vomiting. On examination, she is dehydrated, and her abdomen is
distended. There is a 3 cm x 4 cm swelling in the right groin, which is nontender, and there is
no cough impulse. At operation, a femoral hernia is found. Which one of the following lies
immediately lateral to the hernial neck?
A. Femoral artery
B. Lacunar ligament
C. Pectineal ligament
D. Inguinal ligament
E. Femoral vein

13. A 1-year-old girl with a minimally displaced spiral fracture to her tibia, what is the best
management to be done?
A. External fixation
B. Cast from groin to foot
C. Intramedullary nailing
D. Internal fixation with plates
E. None of the above

14. A 30-year-old patient came with sub capital fracture managed by closed reduction, with
excellent prognosis and follow-up, came after 1 year with pain and hip stiffness. What is the
diagnosis?
A. Avascular necrosis
B. malunion
C. Nonunion
D. Osteoarthritis
E. None of the above

15. A 10-year-old boy fall while playing on his outstretched hand, on examination he minimal
pain and swelling at site of injury, what type of fracture to be found on x-ray?
A. Transverse
B. Comminuted
C. Oblique
D. Spiral
E. Greenstick

16. A young male presented with first metatarsal pain, X-ray showed reduced joint space and
subchondral cysts, what's the diagnosis?

32
A. Pseudo gout
B. Rheumatoid arthritis
C. Gout
D. Osteoarthritis
E. Avascular necrosis

17. A diabetic patient underwent a below knee amputation, then developed a swollen stump,
shortness of breath and chest pain, what is the most likely diagnosis?
A. Pulmonary embolism
B. Gas gangrene
C. Myocardial infarction
D. Pneumonia
E. Cardiac tamponade

18. A diabetic patient underwent a below knee amputation, but the patient developed fever,
swollen red stump and crepitus. What is your provisional diagnosis?
A. Anthrax
B. DVT
C. Gas gangrene
D. Cellulitis
E. None of the above

19. A 50-year-old lady with a long bone fracture, has low Ca level and high ALP. what is the
diagnosis?
A. Paget’s disease
B. Osteoporosis
C. Osteomalacia
D. Metastatic bone tumor
E. Rickets

20. A 75-year-old man presented with backpain for 1 month and irregular urine stream. On
PR examination he has hard prostate mass, his PSA is more than 300. What is the first line of
treatment regarding his state?
A. Hormonal therapy
B. Radiotherapy
C. Watchful waiting
D. Radical prostatectomy
E. Orchiectomy

21. A patient with absent dorsalis pedis and posterior tibial artery pulses after RTA. He has no
sensation below knee, and his leg was internally rotated. What is the likely cause?
A. Neck of femur fracture
B. Posterior hip dislocation with sciatic nerve injury
C. Anterior hip dislocation

33
D. Fracture distal femur
E. None of the above

22. A 25 years old athlete came with a single rib fracture. He is in pain, but he is vitally stable.
What is the most appropriate management?
A. Chest drain
B. Admission, observation and analgesia
C. Analgesia and discharge
D. Infiltration of wound with 5% bupivacaine
E. None of the above

23. A 59 years old COPD patient came with a rib fracture. X-ray was done with no hemo or
pneumothorax detected, but the patient is in pain. What is the most appropriate mgt?
A. Chest drain
B. Admission, observation and analgesia
C. Analgesia and discharge
D. Infiltration of wound with 5% bupivacaine
E. None of the above

24. A 34 years old patient presented after chest trauma. The trauma affected the ribs from
5th to 10th which were fractured. The patient is cyanosed and tachycardic. What is the most
appropriate management?
A. Chest drain
B. Admission, observation and analgesia
C. Analgesia and discharge
D. Infiltration of wound with 5% bupivacaine
E. None of the above

25. A 32 years old athlete patient presented with restrictive movement of whole spine at all
regions. He denies any history of trauma. What is the most likely cause?
A. Ankylosing spondylitis
B. Spondylolisthesis
C. Spondylolysis
D. Spinal canal stenosis
E. Spina bifida

26. A 45 years old male presented with frequency and urgency. On cystogram and after
voiding, he feels incomplete emptying. He was found to have increased intravesical pressure
with residual volume of 200ml. What is the most likely cause?
A. BPH
B. Bladder outlet obstruction
C. Bladder diverticulum
D. Urethral stenosis
E. Underactive bladder

34
27. A 49 years old female has nephrostomy for infected hydronephrosis 3 months ago. She is
now undergoing hysterectomy for a fibroid. What is the best investigation for follow up of
hydronephrosis?
A. Antegrade urethrogram
B. Retrograde urethrogram
C. CT abdomen
D. MRI
E. USS

28. The most common reported infection transmitted by needle prick is:
A. Hepatitis C
B. Hepatitis B
C. HIV
D. Herpes simplex virus
E. CMV

29. A 6-year-old boy brought by his mother with a scrotal swelling which disappears on lying
down. What is the most likely cause?
A. Patent processus vaginalis
B. Undescended testis
C. Direct inguinal hernia
D. Epidydimal cyst
E. None of the above

30. An 8 years old child presented with right leg fracture. Onion peal appearance was
detected in xray. What is the most likely diagnosis?
A. Stress fracture
B. Osteosarcoma
C. Multiple myeloma
D. Ewing sarcoma
E. Paget’s disease of bone

31. A young lady suddenly fell after feeling a sudden pain in the back of her ankle while
walking. What is the test to be positive?
A. Thomas test
B. Phalen test
C. Simmonds Thompson test
D. Froment test
E. Retropulsion test

32. A 35 years old female sustained urinary incontinence obstructed labor. Which nerve root
is responsible?
A. S1, S2, S3

35
B. S2, S3, S4
C. S3, S4, S5
D. L5,S1,S2
E. L4,L5,S1

33. A 53 years old man presented with painless hematuria and weight loss. Investigations
revealed a urinary bladder mass. What is the most probable histological finding you would
expect?
A. Urothelial cell carcinoma
B. Squamous cell carcinoma
C. Adenocarcinoma
D. Signet ring carcinoma
E. None of the above

34. What is the motor nerve supply of dorsal interosseous muscle of the ring finger?
A. Recurrent median nerve
B. Anterior interosseus nerve
C. Deep branch of radial nerve
D. Posterior interosseus nerve
E. Deep ulnar nerve

35. An athlete suffered RTA resulting in a medially rotated thigh with a large swelling at the
midthigh. What is the most likely underlying diagnosis?
A. Neck of femur fracture
B. Posterior hip dislocation with sciatic nerve injury
C. Anterior hip dislocation
D. Fracture shaft femur
E. None of the above

36. A patient is unable to medially rotate the thigh. What is the affected nerve?
A. Superior gluteal nerve
B. Inferior gluteal nerve
C. Femoral nerve
D. Sciatic nerve
E. Pudendal nerve

37. A 53 years old man fell on his left forearm. On examination, there’s numbness over the
first dorsal web space of hand. Which nerve is involved?
A. Posterior interosseus nerve
B. Superficial ulnar nerve
C. Radial nerve
D. Palmar branch of median nerve
E. Anterior interosseus nerve

36
38. A 53 years old diabetic hand worker presented with a fixed bent left ring finger. Extending
the finger produces a popping sound. What is the most likely diagnosis?
A. Rheumatoid arthritis
B. Carpal tunnel syndrome
C. Trigger finger
D. Heberden disease
E. None of the above

39. What is the spinal route value of tibial nerve?


A. L4-S3
B. L2-4
C. S2,3,4
D. L3,4
E. L2,3

40. In popliteal fossa, what is accurate relation of popliteal artery?


A. Most lateral, most deep
B. Most medial, most superficial
C. Most medially, most deep
D. Most lateral, most superficial
E. None of the above

41. A 45-year-old man presents with backache and leg pain due to a prolapsed lumbar
intervertebral disc. The pain, which is aggravated by coughing and sneezing, radiates to the
lateral aspect of the foot. On examination, there is weakness of the plantar flexors of the
foot, which nerve root is most likely to be involved?
A. L5
B. L4
C. S1
D. L3
E. None of the above

42. A 42 years old woman has a cholecystectomy and develops a self-limiting postoperative
wound infection. By what process would bacterial ingestion have been enhanced?
A. Apoptosis
B. Autophagy
C. Metaplasia
D. Opsonization
E. Phagocytosis

43. A 58-year-old woman presented to the Emergency Department with a large fluctuant
swelling at the site of a recent insect bite. She is anxious, tachycardic, and pyrexia. ECG shows
critical fibrillation. She is noted to have a goiter. The swelling at the site of the bite requires

37
surgical drainage. Which of the following classes of drug would be most appropriate as part
of her preoperative preparation for surgery?
A. Alpha-1 adrenoceptor agonist
B. Alpha-2 adrenoceptor agonist
C. Alpha adrenoceptor blocker
D. Beta adrenoceptor agonist
E. Beta adrenoceptor blocker

44. A 45-year-old man has established cirrhosis. At a follow-up appointment a palpable


spleen four fingers breadths below the costal margin is noticed. Full blood counts have shown
a persistent thrombocytopenia. Bone marrow examination shows megakaryocyte
hyperplasia. What is the most likely cause of the thrombocytopenia?
A. Ineffective production in the bone marrow
B. Platelet destruction in the bone marrow
C. Platelet destruction in the liver
D. Platelet destruction in the spleen
E. Platelet storage in the spleen

45. Patient with graves' disease came after 12 months with relapse. what to give her now?
A. Propylthiouracil
B. Propranolol
C. Steroids
D. Carbimazole
E. None of the above

46. A young man stabbed in the Left 5th ICS, he is vitally stable apart of tachycardia, normal
chest x-ray, what is the next step to do?
A. Echocardiogram
B. Ct scan
C. thoracotomy
D. insert chest drain
E. MRA

47. An 18-year old woman recently started the combined oral contraceptive pill and present
with acute abdominal pain, vomiting and muscle weakness on examination she has a pulse
rate of 86beat/minute and her blood pressure is 160/85 mmHg other findings are:
Result Normal
Serum amylase 110 IU/L <100
Serum ALT (alanine transferase) 22 IU/L <50
Alkaline phosphatase 102 IU/L 50-120
Albumin 41 g/L 35-50
Gamma GT 37 IU/L <60
Bilirubin 10 mol/L 0-20

38
calcium 2.41 mmol/L 2.20-2.70
Plasma glucose 5.0 mmol/L
Triglycerides 1.5 mmol/L <1.7
What is the most likely diagnosis?
A. Acute intermittent porphyria
B. Diabetic ketoacidosis
C. Hypoparathyroidism
D. Mumps
E. Pancreatitis

48. A 20-year-old woman presents acutely with abdominal pain. Imaging reveals a right
ovarian cystic lesion which is excised. Histological examination shows a cyst lined by
keratinizing stratified squamous epithelium. Areas of fat, muscle, thyroid and neural tissue
are seen in the wall. What is the appropriate pathological designation for this lesion?
A. Cystadenoma
B. Dysgerminoma.
C. Mesenchymoma
D. Squamous cell carcinoma
E. Teratoma

49. A 33-year-old lady with breast implant and she was worried as she has a strong family
history of breast carcinoma, how to screen for breast cancer?
A. MRI
B. US
C. Mammography
D. FNAC
E. None of the above

50. A 23-year-old girl presented with lump in her right breast. Investigation for this lump?
A. Mammography
B. U/S
C. MRI
D. X-RAY
E. FNAC

51. A 33 years old lactating female presented with breast swelling and redness not
responding to antibiotics. what is your provisional diagnosis?
A. Inflammatory breast malignancy
B. Cellulitis
C. Lactational mastitis
D. Duct ectasia
E. Breast T.B

52. Which of the following is a breast cancer marker?

39
A. CEA
B. CA 72-9
C. HIAA
D. Chromogranin A
E. None of the above

53. A 33 years old patient had splenectomy. What to give her as post splenectomy vaccines?
A. Streptococcus pneumonia vaccine.
B. Streptococcus pneumoniae, Hemophilus influenzae type B vaccines.
C. Streptococcus pneumoniae, Hemophilus influenzae type B, and Neisseria meningitides
vaccines
D. Streptococcus pneumoniae, Hemophilus influenzae type B, and Neisseria meningitides
Vaccines and penicillin
E. None of the above

54. A Chinese lady has conductive hearing loss, palatine ulcer and cervical lymphadenopathy,
most likely diagnosis could be?
A. Nasopharyngeal carcinoma
B. Oropharyngeal carcinoma
C. TB
D. Infectious mononucleosis
E. Maxillary sinusitis

55. A 9-year-old infant presented with fever and tender red mass below angle of mandible
what is your diagnosis?
A. Submandibular abscess
B. Branchial cyst
C. Sebaceous cyst
D. Submandibular gland tumor
E. None of the above

56. A 66 years old patient has lung cancer, during his metastasis work up, there were brain

Metastatic lesions. What is the best drug to do for him?


A. Dexamethasone
B. Radiotherapy
C. NSAID
D. Morphine
E. Mannitol

57. A previously fit 21-year-old man collapses while running a Marathon. Resuscitations
unsuccessful a pos-mortem examination shows an intracranial hemorrhage. what is the most
likely site of the hemorrhage?
A. Intraventricular hemorrhage

40
B. Chronic sub Dural bleed
C. Acute sub Dural bleed
D. Extra Dural hemorrhage
E. Subarachnoid hemorrhage

58. A patient hit by stick, has depressed skull fracture, with no vomiting or loss of
consciousness. What is the next step?
A. CT within 2 hours
B. CT within 8 hours
C. call neurosurgeon
D. admission under observation
E. craniotomy

59. A patient came after RTA, his GCS was less than 10, after log rolling, he suddenly became
hypoxic with rapid decrease in po2 what is your explanation?
A. Respiratory center depression
B. Spinal cord injury
C. Tracheal tube displacement
D. Pneumothorax
E. Large amount of hemorrhage

60. Post splenectomy, what is the blood component affected firstly in CBC?
A. Platelets causing thrombocytosis
B. Platelets causing thrombocytopenia
C. Reticulocytes
D. Eosinophils
E. Granulocytes

61. A 45-year-old homeless man presents with a cough and weight loss over three months.
On examination his BMI (Body Mass Index) is 19 and he has reduced breath sounds in the
right upper zone. His chest X-ray shows a caveating lesion in the right upper lobe. He
undergoes a bronchoscopy and a bronchial biopsy. The biopsy shows featureless necrosis
surrounded by epithelioid macrophages and giant cells. Which of the following is the most
likely diagnosis?
A. Actinomycosis
B. Bronchiectasis
C. Sarcoidosis
D. Squamous cell carcinoma
E. Tuberculosis

62. A 30-year-old woman presents to the general surgical clinic with a 1.5 cm cervical lymph
node along the anterior border of sternocleidomastoid muscle. Clinical examination and
routine blood tests are unremarkable. She undergoes an excisional biopsy of the lump as a

41
day case. The histology report reveals encapsulated infiltrative carcinoma with marked
fibrosis and cystic changes within the lymph node. What is the most likely primary pathology?
A. Papillary thyroid carcinoma
B. Follicular thyroid carcinoma
C. Anaplastic thyroid cancer
D. Lymphoma
E. Medullary thyroid cancer

63. A 22-year-old man has been stabbed in the left fifth intercostal space at the edge of the
sternum. Which structure is most likely to have been penetrated?
A. Left atrium
B. Left lobe of the liver
C. Left ventricle
D. Right atrium
E. Right ventricle

64. A 30-year-old man is admitted to the intensive care unit with an isolated severe head
injury. A CT scan shows multiple intracerebral bleeds but no midline shift. He is intubated and
ventilated. His pupils are dilated and react sluggishly to light. His heart rate is 50
beats/minute blood pressure 170/110 mmHg and his respiratory rate is set at 10
breaths/minute. The rising blood pressure is likely to be caused by:
A. aortic and carotid baroreceptor stimulation
B. Cortisol stimulation
C. renin-angiotensin stimulation
D. sympathetic stimulation related to blood loss
E. sympathetic stimulation related to intracranial pressure

65. A 62-year-old woman presents to her General Practitioner with a two-week history of
back pain. She has lost 8 cm in height over the last five years Investigations revealed Findings:
Results Normal
Corrected calcium 2.78 mmol/L 2.15- 2.55 mmol/L
Phosphate .84 mmol/L .8 -1.4 mmol/L
GFR .96 ml/min More than 90
PTH 8.9 pmol/L .95 -5.7

A. Secondary hyperparathyroidism
B. Tertiary hyperparathyroidism
C. Hypoparathyroidism
D. Osteoporosis
E. Primary hyperparathyroidism

66. The right and left pulmonary arteries are derived from which of the following
embryological aortic arches? (ductus arteriosus as well-new)
A. Second aortic arch
42
B. Third aortic arch
C. Fourth aortic arch
D. Fifth aortic arch
E. Sixth aortic arch

67. Activation of which of the following transport systems best describes how aldosterone
leads to maintenance of the intravascular volume and oliguria?
A. Na+/glucose in the proximal tubule
B. Na/H+ in the descending loop of Henle
C. Na+/K in the ascending loop of Henle
D. Na+/Cl in the distal tubule
E. Na/K in the collecting ducts

68. A 69-year-old man has been admitted to the high dependency unit following an anterior
resection under general anesthesia. He was given 2 mg of intrathecal morphine. On
examination, he looks pale and drowsy. Arterial blood gasses results are:
PH PCO2 PO2 Base excess Glucose Lactate
Findings 7.28 8.1 kPa 10.2 kPa 2.1 20mmol/L 4mmol/L
Normal 7.35-7.45 4.7 - 6.0 11.9 – 13.3 -2 to +2 4-7 0.5 – 2.2

Which of the following is the most likely diagnosis?


A. Diabetic ketoacidosis
B. Metabolic acidosis
C. Metabolic alkalosis
D. Respiratory acidosis
E. Respiratory Alkalosis

69. Pericardio-peritoneal membrane defect will lead to:


A. Congenital Diaphragmatic hernia
B. Ectopia cordis
C. Hiatus hernia
D. Eventration of diaphragm
E. Dextrocardia

70. Facial nerve transection during parotidectomy will lead to all of the following except:
A. numbness over the cheek
B. loss of corneal reflex
C. lower lip dropping
D. drop angle of the mouth
E. none of the above

71. Post parotidectomy, a patient developed gustatory sweating during meals. What is the
mechanism?

43
A. Regeneration of the parasympathetic fibers of auriculotemporal nerve into sympathetic
fibers innervating the sweat gland
B. Regeneration of sympathetic fibers on the parasympathetic of auriculotemporal nerve
C. Regeneration of facial nerve after its injury
D. Regeneration of great auricular nerve on sympathetic fibers innervating the sweat gland
E. None of the above

72. 32 years old patient with pancreatitis. Which enzyme is responsible for autodigestion?
A. lipase
B. elastase
C. trypsin
D. amylase
E. pepsin

73. A 26-year-old man is admitted to the Emergency Department with multiple peripheral
fractures. He is clinically shocked. Which is the structure responsible for the first hemostatic
response to a fall in systemic arterial blood pressure?
A. Adenohypophysis
B. Baroreceptor
C. Chemoreceptor
D. renin
E. Neurohypophysis

74. Failure of caudal part of metanephros to develop will result in:


A. renal agenesis
B. horseshoe kidney
C. polycystic kidney
D. ureter agenesis
E. double ureter

75. Which of the following is required to be divided during tracheostomy?


A. Anterior jugular vein
B. Thymus
C. Thyroid isthmus
D. Recurrent laryngeal
E. Ascending pharyngeal

76. The branchial cyst is derived from which pharyngeal arch?


A. First
B. Second
C. Third
D. Fourth
E. Sixth

44
77. Which of the following organs has a lymphatic drainage to para-aortic LN?
A. Ovary
B. Scrotum
C. Cervix
D. Anal canal above dentate
E. Anal canal below dentate

78. What is the lymphatic drainage of anal canal below dentate line?
A. Superficial inguinal lymph nodes
B. Deep inguinal lymph nodes
C. Internal iliac lymph nodes
D. Obturator lymph nodes
E. Paraaortic lymph nodes

79. What is the lymphatic drainage of scrotum?


A. Superficial horizontal inguinal lymph nodes
B. Superficial vertical inguinal lymph nodes
C. Deep inguinal lymph nodes
D. Internal iliac lymph nodes
E. Paraaortic lymph nodes

80. What is the posterior relation of the inguinal canal?


A. external oblique aponeurosis
B. internal oblique muscle
C. rectus muscle
D. posterior rectus sheath
E. conjoint tendon and Fascia Transversalis

81. Countercurrent mechanism leading to the concentration of urine in the loop of Henle is
mainly mediated by?
A. Active transport of solute out of thin section ascending limb
B. Impermeability to water in thick ascending limb
C. Permeability to solute in descending limb
D. Permeability to solute in thick ascending limb
E. Permeability to water of thin section ascending limb

82. A 33 years old man presented after RTA with complete transection of spinal cord but still
have bladder control due to:
A. Intact parasympathetic to external urethral sphincter
B. Intact innervation to internal sphincter
C. Intact detrusor muscle innervation
D. Intact pudendal nerve
E. None of the above

45
83. The surface landmarks used to localize the optimal site for a tracheostomy incision are
midway between the suprasternal notch and which of the following?
A. Cricoid
B. Hyoid
C. Mastoid
D. Laryngeal prominence
E. Thyroid

84. A 70-year-old man is admitted to hospital with a 12-hour history of a painful white leg. A
femoral artery embolus is shown on angiography. He is an insulin controlled diabetic and
takes ibuprofen for longstanding osteoarthritis. Twenty-four hours after emergency
embolectomy his blood pressure is 90/60 mmHg and he passes very dark brown urine. A
dipstick shows myoglobin and traces of blood. His blood glucose is 15 mmol/L (normal- 4.0-
6.0), urea 12 mmol/L (normal 3.2-7.5) and creatinine 180 mmol/L (normal 35-110). What is
the most likely cause of his abnormal renal function?
A. Drug induced nephropathy
B. Hypovolemia
C. Methemoglobinemia
D. Rhabdomyolysis
E. Uncontrolled diabetes

85. Regarding myasthenia gravis, receptors to be affected is related to:


A. acetylcholine post synaptic cholinergic receptor
B. adrenaline
C. norepinephrine
D. dopamine
E. GABA

86. What is the first branch of internal carotid artery?


A. Ophthalmic artery
B. Anterior cerebral artery
C. Middle cerebral artery
D. Posterior cerebral artery
E. Middle meningeal artery

87. A baby was diagnosed with tetralogy of Fallot, which of the following is associated with
this condition?
A. Pulmonary stenosis
B. ASD
C. Left ventricular hypertrophy
D. Left to right shunt
E. All of the above

88. The most common site of obstruction in hydrocephalus?

46
A. Foramen of Monro
B. Foramen of Luschka and Magendi
C. Cerebral aqueduct of Silvius
D. Subarachnoid space
E. None of the above

89. Site of adrenaline effect to increase heart rate is:


A. B1
B. B2
C. α1
D. α2
E. None of the above

90. Left cardiac shadow in chest x-ray is mainly composed of?


A. Left Ventricle
B. Right ventricle
C. Left atrium
D. Right atrium
E. None of the above

91. A healthy 36 years old man is being assessed with a view to be a life related kidney donor.
Which of following investigations is the most accurate for measuring the GFR?
A. Creatinine clearance
B. Glucose Clearance
C. Inulin Clearance
D. PAH
E. Urea clearance

92. What is the main structure to be divided during Pfennensteal incision?


A. Posterior rectus sheath
B. External oblique muscle
C. Internal oblique muscle
D. Conjoint tendon
E. Rectus muscle

93. According to the UK law (or the GMC), if someone had blood transfusion before,
A. he will not be able to donate before 6 months
B. will not be able to donate before 1 year
C. will not be able to donate forever
D. will be able to donate only if his viral screening is negative
E. None of the above.

94. A patient with Cushing disease, the most common lab abnormality to be seen is:
A. hyperkalemia

47
B. hypokalemia
C. hyponatremia
D. hypocalcemia
E. hypomagnesemia

95. After extensive burn, a patient presented with painless abdominal distension associated
with hiccup and vomiting?
A. Curling ulcer
B. Acute gastric dilatation
C. Sepsis
D. Intestinal obstruction
E. Pancreatitis

96. A 50-year-old man with progressive cough, dyspnea and weight loss starts to complain of
polyuria. His serum calcium is high, on X-ray he has lung mass, invest. to be done is:
A. ΡΤΗ
B. 1,25 cholehydroxycalciferol
C. ACTH
D. Parathyroid hormone related peptide
E. Histamine

97. A patient has developed iatrogenic perforation of esophagus during endoscopy session,
what is the suitable method of nutrition regarding his condition?
A. TPN
B. PEG
C. NGT
D. Jejunostomy
E. Elemental diet

98. Young Lady has many diseases prepared for surgery, which of the following scores is not
relevant in the perioperative period?
A. ASA
B. POSSUM
C. Charlson
D. APACHE
E. None of the above

99. 58-year-old woman, a known patient with thoracic right-sided meningioma, presents with
features suggestive of a Brown-Sequard syndrome. The clinical findings will include which of
the following?
A. Left sided weakness, left sided proprioception and vibration loss, right sided loss of pin prick
sensation
B. Right sided weakness, right sided proprioception, and vibration loss, left sided loss of pin
prick sensation

48
C. Right sided weakness, right sided proprioception and vibration loss, right sided loss of pin
prick sensation
D. Right sided weakness, left sided proprioception and vibration loss, right sided loss of pin
prick sensation
E. left sided weakness, right sided proprioception and vibration loss, left sided loss of pin prick
sensation.

100. A patient was admitted after RTA, undergone splenectomy, developed petechial rash,
has low PLT, HB, fibrinogen and high PT and APTT. What is your diagnosis?
A. DIC
B. idiopathic thrombocytopenic purpura
C. Sepsis
D. Aplastic anemia
E. Herpes zoster

101. An Elderly man was planned for GI surgery and asked about the importance of taking
carbohydrate drink before the operation as part of enhanced recovery program
A. to reduce insulin resistance and improve nitrogen balance
B. to treatment of Addisonian crisis
C. to prevent diabetic ketoacidosis
D. to prevent hypothermia
E. none of the above

102. An Elderly man has bilateral femur shaft fracture underwent fixation, intra operative
blood loss was 800ml, after 1-hour post op he became tachypneic and then arrested:
A. Fat embolism
B. Tension pneumothorax
C. PE
D. Acute left ventricular failure
E. ARDS

103. Patient with multiple small bowel resection due to strictures, the best type of nutritional
support
A. TPN
B. NGT
C. PEG
D. Jejunostomy
E. Open gastrostomy

104. Hypospadias is a result of defect related to:


A. urogenital fold
B. urogenital tubercle
C. patent urachus
D. mesonephron

49
E. none of the above

105. Epispadias is a congenital anomaly:


A. Opening of urethra on anterior surface of penis
B. Opening of urethra on inferior surface of penis
C. Due to abnormal positioning of the genital tubercle
D. Associated with bladder exstrophy
E. None of the above

106. Which of the following is a sign of anxiety before surgery?


A. Bronchodilation
B. Micturition
C. Mitosis
D. Increased peristalsis
E. None of the above

107. A child was brought by his mother due to painless fresh rectal bleeding of small amount
not associated with abdominal pain or change in abdominal habit. What is the diagnosis?
A. Intussusception
B. Juvenile polyp
C. Diverticulum
D. Anal fissure
E. None of the above

108. A patient with hypothermia 35.8C is going to elective hernia repair. What is the
appropriate management?
A. do not operate before core temperature of 36
B. intraperitoneal lavage and proceed
C. warm intra-venous fluid to the patient and proceed
D. raise temperature of the room
E. None of the above

109. A patient came after RTA with head injury, his Glasgow coma scale dropped. What are
you going to expect regarding his heart rate and blood pressure?
A. blood pressure increases, heart rate decreases
B. blood pressure increases, heart rate increases
C. blood pressure decreases, heart rate increases
D. blood pressure decreases, heart rate decreases
E. None of the above

110.which of the following reduce postoperative risk of infection?


A. Shaving immediately before operation
B. Skin sterilization with iodine
C. Theater zooning

50
D. Preoperative antibiotic
E. Contact Ulcer

111.A patient with antithrombin 3 deficiency, to be treated with:


A. LMWH
B. Unfractionated heparin
C. warfarin for life
D. warfarin for 6 months
E. stocking

112. A 53 years old lady with metastasis symptoms and back pain, with hypercalcemia. What
to be administered first?
A. Prednisolone
B. 0.9% Normal Saline
C. Pamidronate
D. Zoledronate
E. I.V K supplementation

113. A man suffered a gunshot through the right 9th intercostal cartilage, the most likely
organ to be injured is:
A. Liver
B. Fundus of gall bladder
C. Body of gall bladder
D. Left Kidney
E. Pancreas

114. Parts of conducting system of heart include which of the following?


A. SA node
B. Musculi pectinati
C. AV node
D. Chorda tendenie
E. None of the above

115. Uses of PAHA (Para Amino Hipruric Acid) include which of the following?
A. estimation of renal blood flow
B. estimation of GFR
C. estimation of urine output
D. estimate the filtration fraction
E. None of the above

116.Branchial cyst is a derivative of which arch?


A. 1st pharyngeal
B. 2nd pharyngeal
C. 3rd pharyngeal

51
D. 4th pharyngeal
E. 6th pharyngeal

117.Dartos muscle is layer of scrotum derived from?


A. Superficial fascial layer.
B. Cremasteric muscle
C. Internal oblique muscle
D. Peritoneum
E. External oblique muscle

118.A Pigmented gall bladder stone is mainly composed of:


A. Calcium oxalate
B. Calcium bilirubinate
C. Cholesterol
D. Bile salts
E. None of the above

119.Causes of hemolysis in splenomegaly include which of the following?


A. Destruction by splenic sinusoids (sequestration)
B. Secretion of autoimmune antibodies
C. Portal hypertension
D. Bone marrow hyperplasia
E. None of the above

120.Structure passing through the diaphragmatic opening at T12 include:


A. Azygos vein and thoracic duct
B. Vagus nerve and esophagus
C. IVC and phrenic nerve
D. Ureter and gonadal vessels
E. None of the above.

121.Which statement is true about right coronary artery origin?


A. Posterior from aorta above aortic valve
B. Anterior from aorta above aortic valve
C. Posterior from aorta below aortic valve
D. Anterior from aorta below aortic valve
E. None of the above

122.What are the boundaries of the correct site for intercostal chest drain insertion?
A. Pectoralis major, serratus and line through the nipple
B. Pectoralis minor, serratus and line through nipple
C. Pectoralis major, latissimus dorsi and line through the nipple
D. Pectoralis minor, latissimus dorsi and line through nipple
E. None of the above

52
123. Support of cervix is mainly by:
A. Broad ligament
B. Ovarian ligament
C. Round ligament
D. transverse cervical ligament
E. none of the above

124.Hypothermia is identified on ECG by:


A. Delta wave
B. U wave
C. J wave
D. shortened PR
E. None of the above

125.After salivary gland surgery, salivation was impaired. What is the involved ganglion?
A. Pterygopalatine ganglion
B. Submandibular ganglion
C. Gasserian ganglion
D. Ciliary ganglion
E. None of the above

126.A 6-year-old child requires a long-term drug therapy to treat a rare genetic disorder. The
drug must be given intravenously. Recently, there have been issues with the child pulling the
currentHickman line and the parents are requesting an alternative. What is the best option?
A. Triple lumen subclavian line
B. Porta Cath device
C. Intermittent cannulation as needed
D. PICC line
E. Broviac line

127.A patient with symptoms of rheumatoid arthritis. What is the most specific factor for his
condition?
A. ANCA
B. ANA
C. Antimitochondrial antibody
D. Antiphospholipid antibodies
E. Antimicrosomal antibody

128.A pregnant tall lady at 36-week gestation came with a sudden sharp chest pain, her
father died from MI, what is the most likely diagnosis?
A. Aortic dissection
B. Pulmonary embolism
C. Myocardial infarction

53
D. Pericarditis
E. None of the above

129. You are processing a clinical study on a group of 87 patient about giving IV fluids and
postoperative dehydration. It depends on reading urea and creatinine before and during
operation. The collected data was normally distributed. What is the statistical test you will
use?
A. Paired T test
B. Mann Whitney U test
C. Chi-square
D. Unpaired T test
E. Wilcoxon

130. A study comparing fluid requirements in burn patient, study participants are classified to
males and female groups, most patient had small values and some larger values. Which test
to apply?
A. Paired T test
B. Unpaired T test
C. Mann Whitney U test
D. Chi-square
E. None of the above

131.What is Reciprocal of absolute risk reduction?


A. Number needed to treat.
B. Relative risk reduction.
C. Odds ratio.
D. Sensitivity.
E. Probability.

132.A 45-year-old patient presented with abdominal pain, flushing and diarrhea, underwent
appendectomy with histological finding of 4 cm carcinoid with free surgicalmargins. What is
the best next investigation?
A. CT scan abdomen with contrast
B. Urinary 5-HIAA
C. Abdominal USS
D. MRI Abdomen and pelvis
E. PET-Scan

133.What is the best method for Sterilization of endoscopy tools?


A. Plasma sterilization
B. Alcohol 70%
C. Glutaraldehyde
D. Gamma radiation
E. Heat Oven

54
134.A 52 years old smoker with large venous ulcer on her medial malleolus for around 30
years, which of the following could be due to long-term varicose veins?
A. Dark discoloration of the toes
B. Eczema around the ankle
C. Foot ulceration
D. Amputation of toes
E. Loss of pulsation

135. A 50-year-old patient with Barrett disease for a long time, presented now complaining
from 2 months history of dysphagia and weight loss, histological finding of esophagus will
reveal?
A. Squamous cell carcinoma
B. Adenocarcinoma
C. Signet ring carcinoma
D. Aden squamous carcinoma
E. Columnar metaplasia

136. Male 35 years old presented with tender fluctuate perianal and rectal swelling, he is
feverish for that past 2 days and cannot tolerate PR examination. The most appropriate
treatment is:
A. Incision of this swelling and primary closure
B. Excision of this swelling
C. Hemorrhoidectomy
D. Medical Treatment
E. Incision of the swelling and lay open

137. Male patient 45 years old complained from a long history of upper abdominal pain due
to chronic use of analgesics, now he presented with sever agonizing abdominal pain and you
suspect that he has perforated duodenal ulcer. What is the best investigation modality to
detect free air in this abdomen?
A. CT Abdomen pelvis with contrast
B. CT Abdomen pelvis without contrast
C. Barium swallow
D. Erect chest-abdomen x-ray
E. Water soluble contrast

138.A 45-lady started to complain from right leg pain and swelling, after traveling from japan
to UK, her peripheral pulses are intact, ultrasound revealed a large femoral venous thrombus,
what is to be given immediately?
A. Oral Warfarin
B. IV Heparin
C. Thrombolysis
D. Aspirin

55
E. IVC Filter

139.A 50-year-old patient dealing with sheep presented with weight loss, hepatomegaly
which is tender on examination. MRI liver show a well round calcifications inside the liver,
with eosinophilia in FBC. What is your diagnosis?
A. HCC
B. Pyogenic liver Abscess
C. Hydatid disease of the liver
D. Cholangiocarcinoma
E. Liver Mets

140.A 65-year-old patient with ulcerative colitis complaining from peritonitis and bowel
perforation. Surgeons decide to perform an ileostomy. After 6 months he developed skin
ulceration around stoma site. what is the name of these ulcers?
A. Erythema nodosum
B. Pyoderma gangrenosum
C. Necrotizing fasciitis
D. Squamous cell carcinoma
E. Contact Ulcer

141.A 35 years old lady with recurrent episodes of RUQ colic pain, USS showed multiple small
gall stones with increase thickness of the wall with normal CBD, what to do next?
A. Elective cholecystectomy
B. MRCP
C. Urgent cholecystectomy
D. Liver function tests
E. ERCP

142.8-year-old boy with chronic constipation for a long period and he presented now
complaining
from a very agonizing pain from the anus associated with brisk fresh blood at the end of
defecation, what is your provisional diagnosis?
A. Juvenile polyp
B. Anal Hemorrhoids
C. Intussusception
D. Anal fissure
E. Perianal abscess

143.A 50-year-old woman presents with a history of right upper quadrant pain and jaundice
which is progressive and unremitting. She reports that her urine was dark in color and that
her stools are offensive and difficult to flush. By examination she has a palpable mass in the
right upper quadrant region. Which of the following is the most likely diagnosis?
A. Chronic calcular cholecystitis
B. Primary biliary cirrhosis

56
C. Gallbladder abscess
D. Carcinoma head of pancreas
E. CBD stones

57
APRIL 2021 RECALLS
1. What is the relationship of the ureter to the gonadal vessels at the (pelvic brim)?
a. Posterior
b. Posterolateral
c. Medial
d. Lateral
e. Anterior

2. A patient has diplopia of the right eye. And he is trying to look to left and downward in
image B- what nerve is affected?

a. Occulomotor
b. Trochlear
c. Abducens
d. Vagus

3. A 15-year-old girl had parathyroidectomy done one year ago. Now complains of abdominal
pain and diarrhea. What is the cause of her diarrhea?
a. Secretin
b. Thyroxine
c. Glucagon
d. Gastrin
e. Calcitonin

4. A patient with Pancoast tumour and now develops horner’s syndrome. Which neuron is
affected?
a. Lateral horn
b. Posterolateral horn
c. Anterior horn
d. Edinger Westphal nucleus

58
5. What is the estimate volume from one cardiac cycle from a healthy man?
A. 0.7ml
B. 7ml
C. 70ml
D. 400ml
E. 700ml

6. Regarding left coronanry artery?


a. posterior to pulmonary trunk
b. lies in the anterior interventricular groove
c. Arises from posterior aortic sinus
d. Arises from below anterior aortic valve
e. Arises from anterior ascending aorta

7. Regarding Right coronary artery.


a. lies in posterior interventricular groove
b. Arise from anterior wall of ascending aorta.

8. Absorption of Na-K in kidney is an example of?


a. diffusion.
b. Osmosis.
c. Active transport.
d. Facilitated diffusion

9. Furosemide works on?


a. PCT
b. DCT
c. Ascending limb
d. Descending limb

10. Benzoflurothiazide works on?


a. PCT.
b. CT
c. Ascending limb
d. Descending limb
e. DCTs

11. Most common disease transmitted via needle prick?


a. HBV
b. HIV
c. HCV
d. HTLV
e. Malaria

59
12. A patient has subcapital fracture of humerus and develop weakness of the deltoid muscle
following nerve injury. What other muscle will be affected?
a. Suprapinatus
b. Infraspinatus
c. Teres major
d. Teres minor
e. Lattisimus dorsi

13. The pathologist found Hassall’s corpuscles during histology. What gland is this found?
a. Thymus
b. Thyroid
c. Parathyroid

14. 25yo male was playing football and sustained trauma to the skull. Now unable to move
half of his face. Where is the lesion in the facial nerve?
a. In stylomastoid foramen
b. In geniculate ganglion
c. In parotid
d. In pons

15. 11. A patient with cyanotic heart disease was diagnosed to have TOF. One of these is a
component of TOF?
a. Patent foramen ovale
b. Transposition of great vessel
c. Overriding of the Aorta
d. Left ventricular hypertrophy
e. Pulmonary stenosis

16. A patient with facial nerve injury in the left parotid. What abnormalities will he have?
a. Ptosis of the left eyelid
b. Drooping of the left lower lip
c. Numbness on the whole face
d. Numbness of the ipsilateral cheek

17. Pharyngeal pouch location ?


a. Below inferior constrictor
b. Through inferior constrictor
c. Between superior and middle constrictor
d. Between middle and inferior constrictor
e. Behind inferior constrictor

18. A patient with Down's syndrome post cholecystectomy has abdominal pain. What is the
appropriate immediate action?

60
a. Epidural analgesia
b. Patient controlled analgesia
c. Immediate bolus IV morphine
d. Oral paracetamol

19. Patient with Fever and loin pain. Investigations shows a stone (size not given) in the
L2/L3, with dilated kidney and ureter. Immediate treatment?
a. Percutaneous nephrolithotomy with jj stent
b. Percutaneous needle nephrostomy
c. Ureteroscopy...
d. ESWL
e. Ureteroscopy + stent insertion

20. Patient who is tired and lethargic. Examination revealed a firm small goiter. What is the
diagnosis?
a. Hashimoto's thyroiditis
b. Graves
c. Papillary Ca
d. Follicular Ca
e. Anaplastic Ca

21. A lady has a mass on the anterior neck that moves with swallowing. Examination reveals a
2cm mass on the gland, and no lymphadenopathy. What is the first initial investigation to
evaluate this mass?
a. FNAC
b. USS
c. Thyroid function

22. Male patient with longstanding asthma. Takes 15mg prednisone daily and needs to
undergo an operation. Most appropriate changes to be made to his steroid dosage?
A. Increase steroid on day of op and continue for three days post op (ans)
B. Increase steroid on day of op and continue for one month post op
C. Decrease dose on day of op
D. Stop steroid preop
E. Continue normal dose of steroid peri-operatively

23. ECG was performed on patient and there was flat P wave, inverted T wave and prolonged
PR interval which electrolytes was disturbed?
a. Hypokalemia
b. Hyperkalemia
c. Hypocalcemia
d. Hypercalcemia
e. Hyponatremia

61
24. Superficial partial 20% burn with 5% full thickness burn. Treatment?
A. STSG (ans)
B. Full thickness graft
C. Flap
D. microvascular flap

25. Embryological origin of inferior parathyroid gland?


a. 3rd pharyngeal pouch
b. 3rd pharyngeal arch
c. 3rd pharyngeal cleft.

26. Patient presents with abominal pain, she was depressed 2 week ago ??
Lab result were Hypercalcemia and phosphate and PTH were normal. She
was given pridnosolne and calcium return to normal what is your diagnosis?
a. A. Hypoparathyrodism
b. B.Milk alki syndrom
c. Sarcoidosis
d. Sjogren’s

27. Patient with Pernicious anaemia taking IM Vitamin B12 3 monthly. She
presents with haematemeis and weight loss. Blood investigation revealed
hypochromic microcytic anaemia. What's the diagnosis?
a. Atrophic gastritis
b. Gastric cancer
c. Duodenal Ulcer

28. A patient swallowed foreign body and a xray of the neck was done. What
is the name of this structure

a. Thyroid
b. Cricoid
c. Hyoid
d. Epiglottis
e. Larynx
62
29. Athlete pain on deep flexion sustained acetabular labral tear.
What investigations?
a. MR rhogram
b. CT rhogram
c. Arthrogram
d. Xray of the hip

30. Pulmonary artery is a derivative of?


a. First aortic arch
b. Second aortic arch
c. Third aa
d. Fourth aa
e. Sixth aa

31. A patient has a laceration on the upper arm down to the elbow exposing the median
nerve and brachial artery. What is their relationship from up down?
a. Lateral anterior medial
b. Lateral posterior medial
c. Anterior lateral posterior
d. Medial anterior lateral

32. After resection of a rectal tumour a patient experiences erectile dysfunction. Which of the
following nerves is most likely to have been damaged in surgery?
a. A Genitofemoral nerve
b. Lumbosacral plexus
c. Pelvic splanchnic nerves
d. Perineal branch of S4
e. Pudendal nerve

33. A patient took long-term broad-spectrum antibiotics, now developed dysphagia. What is
the cause?
a. Candidiasis
b. Achalasia
c. Oesphageal Ca

34. A 50-year-old woman presents with a history of faecal incontinence over the past few
year She had a prolonged and difficult first stage of labor 20 years previously. Physical
examination reveals a relatively lax anal sphincter. Which nerve is likely to have been
damaged in labour?
a. Autonomic nerves to the rectum
b. Genitofemoral nerve
c. Lumbosacral trunk
d. Obturator nerve

63
e. Pudendal nerve

35. A woman returned to UK from China presents with pain on the lower limb. Investigations
shows extensive mobile thrombus. Best immediate or initial treatment?
a. Aspirin
b. Clopidogrel
c. LMWH
d. Thrombolysis

36. 30 yrs old pt involved in rta o/e Bp /80/40 pr 120. had ribs fracture from 5th-10th with in
drawing of the chest. after short time he became unconscious(i think confused or irritable)
oxygen saturation about 74% what is your action?
a. Immediate intubation
b. Infiltration of 0.5% bupivacaine
c. Infiltration of 2% Lidocaine
d. Chest drain in the 4th ics anteriorly (yes anterior was the option)

37. What is the precursor of noradrenaline precursor which acts on β1 in low concentration
and can have activity on α1 in high dose.?
a. Dobutamine
b. Dopamine
c. Adrenaline
d. Noradrenaline

38. What maintains the resting membrane potential in skeletal muscle?


a. K+
b. Na+
c. Ca2+
d. Cle.

39. Failure of development of the caudal portion of the metanephros result in?
a. Horse shoe kidney
b. Renal agenesis
c. Polycystic kidney disease

40. A man who is a wood carver with medial canthal pain, nasal obstruction
and bloody mucosal discharge: Diagnosis?
a. Ethmoid air sinus cancer
b. maxillary sinus cancer
c. Amelloblastoma

41. 6 months old baby, appropriately immunized for age presented with neck
swelling which occurred rapidly over the last 2 weeks. What is the diagnosis?
a. cystic hygroma

64
b. branchial cyst
c. Lymphoma

42. Where does pelvic collection collects mostly?


a. Retropubic
b. Rectouterine
c. Uterosacral
d. Behind the rectum
e. Behind the sigmoid colon

43. A man with villous adenoma, what is the likely electrolyte imbalance?
a. Hyperkalemia
b. Hypokalemia
c. Hypernatremia
d. Hyponatremia
e. Hypocalcemia

44. A patient who had surgery, now presents with fever 36 hours post op.
Urine and wound is clear. What is the diagnosis?
a. Atelectasis
b. Wound infection
c. UTI
d. Bronchopneumonia

43. What structure is responsible for autonomic nervous control?


a. Hypothalamus
b. Thalamus
c. Medulla
d. Globus Pallidus

45. A table with parameters of increased HR, Decreased SVR, and Increased CO. Diagnosis?
a. Septic shock
b. Hypovolemic shock
c. Cardiogenic
d. Neurogenic

46. A 45-year-old man has established post hepatic cirrhosis. At a follow-up appointment, a
palpable spleen four fingers breadths below the costal margin, is noticed. full blood counts
have shown a persistent thrombocytopenia. Bone marrow examination has shown
megakaryocytic hyperplasia. 'What is the most likely cause of the thrombocytopenia?
a. Ineffective production in bone marrow.
b. Platelet destruction in the bone marrow
c. Platelet destruction in the liver
d. Platelet destruction in the spleen

65
e. Platelet storage in the spleen

47. Patient with diarrhoea from clostridial infection, how will you treat?
a. Oral metronidazole
b. Vancomycin
c. Erythromycin
d. Amoxicillin
e. Ceftriaxone

48. Where does Cancer of the Prostate metastasizes to?


a. Bone
b. Lung
c. Liver
d. Brain

49. Where does Colon cancer metastasizes to?


a. Bone
b. Liver
c. Lung
d. Brain

50. 36yo woman with bloody nipple discharge from one duct?
a. intraductal papilloma
b. Ductal carcinoma
c. Duct ectasia.

51. At which site is the most common location of CSF obstruction?


a. Cerebral aqueduct of Sylvius
b. Infundibular recess
c. Interventricular foramen (of Monro)
d. Lateral foramen of fourth ventricle (foramen of Luschka)
e. Median foramen of fourth ventricle (foramen of Magedie)

52. A patient with back pain develops lateral foot numbness, with weakness in dorsiflexion of
the ankle and Hallux. Intact Ankle and knee jerk reflexes. What nerve root is the cause?
a. Compression of L5
b. L3, L4 compression
c. L4, L5

53. 6 years old boy fell on the outstretched hand and sustained mild swellingand little
deformity of the left lower forearm. What is the likely finding?
a. Greenstick #
b. Multifragmented #
c. Transverse #

66
d. Oblique #
e. Spiral #

54. 16-year-old boy was hit by car on the left thigh with a direct impact, what is the likely
finding?
a. Transverse #
b. Multifragmented fracture
c. Oblique fracture
d. Spiral fracture
e. Greenstick #

55. A 25 yo female was crossing the road, and was kicked on leg at the posterior heel and
couldn’t cross and fell, which of these tests would be useful?
a. Thomas
b. Simmonds
c. Mc murray test

56. You are doing a research to compare two methods of DVT prophylaxis after TKR and using
drain which measures blood collection over 24hours.You got a results which are variable from
very little volume to excessive volume. Which of these statistical tests would be appropriate?
a. Paired T test
b. Mann U Whitney
c. Wilcox
d. Chi square
e. Unpaired t test

57. What is the Inverse of numbers to treat?


a. Absolute risk increase
b. Absolute risk reduction
c. Relative risk
d. Odds ratio

58. Seven days post laparotomy patient becomes confused. Nurse calls you that she noticed a
serosanguinous discharge from the wound? What is the likely explanation?
a. Anastomotic leak
b. Abscess collection
c. Wound dehiscence
d. Wound infection
e. Haemolysed haematoma

59. Patient had reversal of loop colostomy after 3 months. Patient now complains of pain,
vomiting. What is the likely cause?
a. Incisional hernia
b. Necrosis

67
c. Adhesion

60. Px has abdominal pain and this was relieved by rubbing and applying warm cloth. The
pain relief via this method was by?
a. c fiber modulation by A beta,
b. c fiber modulation by A delta,
c. a delta modulation by C fibre

61. An otherwise healthy man with RIF pain free and a smooth, regular mass with defined
edges and free of movement with respiration. What is the likely mass?
a. Visceroptotic kidney
b. Meckles diverticulum
c. Appendix mass
d. Mucocele of gall bladder
e. Caecal tumour

62. A teacher who missed her morning meal frequently, swears and sweats a lot in morning
and her behaviour returns to normal after meal. What is the diagnosis?
a. Insulinoma
b. reactive hypoglycaemia

68. What is the lymphatic drainage of the scrotum?


a. Superficial inguinal LNs
b. Deep inguinal LNS
c. Internal iliac LNs
d. External iliac LNs
e. Para-aortic LNs

69. Squamous cell carcinoma at anal margin/below dentate line drains to which lymph nodes
first?
a. Superficial inguinal LNs
b. Deep inguinal LNS
c. Internal iliac LNs
d. External iliac LNs
e. Para-aortic LNs

70. Patient with pruritus and jaundice. Which of these antibodies will be present in the
blood?
a. Anti-microsomal antibody
b. Anti-mitochondrial antibody
c. Anti-reticulin
d. Anti-parietal cell antibody
e. Anti-nuclear antibody

68
71. Deposition of what on the skin is responsible for itching in obstructive jaundice?
a. Bile salts
b. Conjugated Bilirubin
c. Unconjugated bilirubin

72. What is the relation of the left renal vein to the abdominal aorta.?
a. Anterior
b. Posterior
c. Lateral
d. Medial
e. Superior

73. A patient sustained head trauma presented to the A&E conscious. Hewas noted to have
right, fixed and dilated pupil; Few hours later he deteriorated and did not respond and die.
What is the likely cause?
a. Left EDH
b. Right EdH
c. Acute subdural
d. Subarachnoid

74. A patients sustained head injury and presented to the A&E with a GCS of 8. His BP was
190/100, HR was 40, he was already intubated and ready for transfer to the neurosurgical
unit. What is the immediate method to decrease the ICP in this patient?
a. Hyperventilation
b. Dexamethasone
c. Hemicraniectomy
d. Burr hole
e. Infusion with saline

75. Female with 2cm thyroid nodule and no other symptoms. FNAC benign. Best management
of this case? (did not mention that a US was performed).
A. Hemithyroidectomy
B. Discharge
C. Uss

76. Male with perianal pain and discomfort for two days. Soft and fluctuant mass on DRE.
Most appropriate management?
A. Hemorrhoidectomy
B. Medical therapy
C. Incision
D. Excision

77. Bee sting with anaphylaxis, patient has BP of 60/30mmHg Initial treatment?
A. Subcutaneous adrenaline (SC is not an effective method!)

69
B. IV antihistamine
C. IV hydrocortisone???
D. IV fluids
E. Oral antihistamine

78. Ischioanal abscess. What structure can you feel when you palpate the medial wall of the
abscess?
A. Ischial tuberosity
B. Internal sphincter
C. Pudendal canal
D. Levator ani
E. Obturator internus

79. Mother with two-year-old child can't feel his testis. Examination shows one normal
testicle and the other testicle is not palpable. Initial management?
A. Ultrasound
B. HCG stimulation
C. Laparoscopy
D. Inguinal exploration

80. Male with back pain radiating to loin which started while defecating. Pulsatile abdominal
mass. Chest X-ray shows mediastinal widening. BP is normal. Next appropriate management?
A. CT scan
B. Immediate transfer to theatre
C. Ultrasound
D. Immediate EVAR
E. Follow up each 1/2 hour

81. Warfarin inhibits?


A. Antithrombin
B. Prothrombin
C. Fibrinogen

82. Patient with long term Crohn’s disease has abdominal pain and loss of
appetite. Albumin is 29 (low). What feeding?
A. Enteral nutrition
B. High protein diet
C. Elemental diet
D. Central parenteral nutrition
E. Multivitamin supplements

83. Elderly male with fresh blood PR and diarrhoea for two days. Has tenderness on right side
of abdomen. Is taking atenolol for hypertension. Has had episodes where he passes dark red
and bright stool with loose motion. Diagnosis?

70
A. Ischaemic colitis
B. Ulcerative colitis
C. Meckel diverticulum
D. Anal carcinoma
(no diverticulitis in the options given)

84. Patient with lower limb trauma after RTA. Absent dorsalis pedis and posterior tibial
pulses. Site of fracture?
A. Supracondylar femur
B. Midshaft of tibia
C. Lateral malleolus
D. Neck of femur
E. Femoral epicondyle

85. Patient with right open femur fracture after RTA. After initial resuscitation and traction,
he has a cold, numb and pulseless limb. After removal of traction, what is the best action?
A. Surgical exploration of superficial femoral artery
B. CT angiogram
C. Calf fasciotomies

86. Location of McBurney’s point?


A. ⅔ from ASIS to umbilicus
B 1/3 from ASIS to umbilicus

87. Gardener stabbed pulp of finger with plant thorn. Three days later develops
stabbing pain and erythema to side of nail. Organism?
A. Staph. aureus (ans)
B. Strep. pyogenes
C. Anthrax
D. Klebsiella

88. Five days post hernia repair. Tissues surrounding the wound are swollen
and erythematous. Why?
A. Accumulation of bacteria in tissue
B. Increased vascular permeability
C. Hematoma
D. Increase intracellular fluid

89. Young boy with 6cm scalp laceration. Dirt in wound. Jagged edges. Treatment?
A. Flap.
B. Immediate suture
C. excision and leave for secondary intention
D. Wound excision and suture

71
90. Which is true of gastrin?
A. Increases gastric acid secretion
B. It has a trophic effect on gastric mucosa (ans)
C. It is mostly secreted from gastric body
D. Increases secretion of bicarbonate from pancreas
E. PPIs inhibit its production

91. Elderly male with painless frank haematuria. Long history of smoking. Slightly enlarged
prostate. PSA 3.5 (normal), Hb 107. Some other bloods. Diagnosis?
A. Benign prostatic hyperplasia
B. Prostate cancer
C. Bladder cancer
D. Renal cancer

92. 27-year-old female has several episodes of loin pain after drinking three cups of coffee
every morning. Diagnosis?
A. Ureteric calculi
B. Uretero-pelvic junction obstruction (dilt crisis, acute hydronephrosis in chronic
setting)
C. Polycystic kidney disease (?ans; correlation between caffeine and polycystic
kidney

93. Male patient with skull injury. Facial nerve injured. Sustained ipsilateral facial muscle
weakness. No hyperacusis or change in taste. Site of injury?
A. Parotid gland (?ans)
B. Internal acoustic meatus
C. Stylomastoid foramen (near trunk origin associated with head injury. Chorda
tympani and nerve to stapedius arises early in the tympanic part)
D. Middle ear

94. Stab injury at right fourth ICS near sternal border. Structure injured?
A. Left atrium
B. Right ventricle
C. Aorta
D. Right atrium

95. 15cm retroperitoneal mass displacing stomach, spleen and kidney. History of L4/5
prolapse. What is best to find out diagnosis?
A. Radical Excision
B. Incision
C. Needle core biopsy
D. FNAC

72
96. Patient with arm injury associated with difficulty in extending forearm and arm as well as
numbness over posterior aspect of arm up to dorsal aspect of hand. Cause?
A. Radial nerve compression (?ans)
B. Posterior cord of brachial plexus
C. Lateral cord of brachial plexus
D. Medial cord of brachial plexus

97. 16-year-old boy with knee swelling. History of sports injury but no direct trauma to knee.
X-ray shows alternating… with lifted periosteum and soft tissue mass. What's expected under
microscopy?
A. Small round blue cell with t11:22 chromosomal translocation
B. Malignant… polygonal with osteoid formation
C. Epithelial cell
D. Endothelial cells
E. Osteoid osteosarcoma

98. Patient had mid sigmoid cancer removal. Which is best practice regarding ERAS?
A. Call up initially follow up after 30days
B. Remove catheter day one post op (ans)
C. Pre/postoperative opiates for pain
D. Preoperative bowel preparation

99. Patient with 20% second degree burns on leg and hand (hot water burn).
Most important step in management?
A. Secure airway
B. Crystalloids and colloid (ans)
C. IV antibiotics

100. Young woman with sudden onset headache and photophobia due to ruptured berry
aneurysm. Pathology?
A. Congenital weakness of arterial wall (ans)
B. Mycotic
C. Medial cystic necrosis
D. Atherosclerosis

101. Patient with big hands. Has signs of acromegaly. What is the appropriate Imaging?
A. X-ray hands
B. CT skull
C. X-ray skull lateral view
D. X-ray skull anterior view
E. MRI

102. Patient with hyponatraemia and hyperkalaemia. Most appropriate blood investigation?
A. ADH

73
B. ACTH

103. Patient with bilateral hilar lymphadenopathy. FNAC of mediastinal mass shows glandular
cells, pleomorphic nuclei and is negative for many neuroendocrine stains. Diagnosis?
A. Adenocarcinoma
B. Small cell cancer
C. Squamous cell cancer
D. Mesothelioma
E. Hamartoma

104. Direct of these has a direct effect on blood volume after loss of 1L?
A. Renin
B. Angiotensin I
C. Angiotensin II
D. Angiotensinogen

105. 30 yo pt involved in RTA with no head, abdominal or lower limb injury. BP- ; PR-120;
SPO2- 94%; has rib fractures with indrawing of chest from 5th to 10th rib during breathing.
After a short time, he became confused and irritable and oxygen sat dropped to 74%. What is
your action??
A. immediate intubation
B. Analgesia
C. Infiltration with 0.5% Bupivacaine
D. Infiltration with 2% Lidocaine
E. Chest tube insertion anteriorly (Yes it was anterior in the option)

106. Most important and independent prognostic factor in histopathology in breast cancer?
A. Estrogen receptor status.
B. lymph node status
C. Her 2 neu status

107. A patient with venous disease with Uss showing reflux of Great saphenous vein. Which
of these is as a result of the venous insufficiency?
A. Eczema around ankle
B. Ulcers on dorsum of the foot.
C. Dark discoloration of the toes

108. Patient with multiple myeloma. Rectal biopsy shows pink proteinaceous material.
Diagnosis?
A. Amyloidosis (ans)
B.
C.

109. What is used for with frozen section?

74
A. H&E staining
B. Immunohistochemistry
C. Formaldehyde
D. Formalin
E. Direct smear and look under microscopy

110. Where to do lumbar puncture?


A. Level of the iliac crest below the conus medularis (ans)
B. Sacrococcygeal level
C. Level of the iliac crest above the conus medularis

111. Male with diffuse enlargement of thyroid and increased uptake on scan. Thyroid
function tests will likely show?
A. High T3
B. High T4
C. Low TSH (ans)

112. Female with head injury. Post hematoma evacuation she complains of headache and
nausea. Total input 3L, output 1.2L. Na 122 (low), K 3.6, plasma osmolality 240 (low), urine
osmolality 540 (normal). Treatment?
A. Vasopressin receptor agonist
B. Mannitol
C. Bendroflumethiazide
D. Fluid restriction (ans)
E. Aldosterone

113. Post-trauma patient. Intubated and doing well. Becomes hypoxic during transfer/log roll
Cause?
A. Displaced endotracheal tube (ans)
B. Neurogenic shock

114. Patient with bilateral parotid masses and facial palsy. Responds to steroids. Diagnosis?
A. Sarcoidosis (ans)
B. Sjogren syndrome
C. Milk alkali

115. Location of SGLT2 in kidney?


A. PCT (ans)
B. DCT
C. CT
D. Ascending loop
E. Descending loop

75
116. IVDU patient with knee swelling. Aspiration shows gram+ cocci. Lavage and
cephalosporins are given. Collapses two days later. Chest X-ray shows apical lung lesion with
cavitation. Organism?
A. Staph. aureus
B. Strep. pyogenes
C. Peptostreptococcus
D. Pseudomonas aeruginosa

117. HIV+ African female with haemoptysis. Chest X-ray shows haziness in right apical lobe.
Diagnosis?
A. Tuberculosis
B. Pneumonia
C. Aspergillus

118. Sheep farmer with hydatid cyst. Organism?


A. Echinococcus granulosus (ans)
B. Entamoeba histolytica

119. Watercress farmer with jaundice. Organism?


A. Fasciola hepatica
B. Clonorchis sinensis

120. Elderly female post stroke with 99% or total right carotid artery occlusion on duplex
(radiologist can’t say certainly between the two). What next?
A. Endarterectomy
B. Carotid angiogram
C. MRI
D. CT head
E. External to internal bypass

121. Intoxicated student needs to urinate. Runs into a post and loses urge. Next morning
comes into ED with suprapubic pain and no urine output. Bladder not palpable but there is
suprapubic tenderness. Diagnosis?
A. Bladder rupture (ans)
B. Rectus sheath haematoma
C. Urethral rupture
D. Membranous rupture

122. 90-year-old female with erythema and ulceration of nipple. Diagnosis?


A. Paget disease of the nipple (ans)
B. DCIS

76
123. Female with nipple destruction extending to areola. Besides a mammogram, what other
investigation would you like to do to confirm diagnosis? (Punch biopsy was not in the
options)
A. Skin biopsy from areola (ans)
B. Trucut biopsy
C. FNAC
D. Incisional biopsy
E. Exfoliative cyotology

124. High riding prostate cause?


A. Membranous urethral rupture
B. Prostatic urethra rupture
125. Depolarising neuromuscular blocker?
A. Suxamethonium

126. 20-year-old male hit in groin a couple of weeks ago. Six weeks later has painless irregular
hard testicular swelling that transilluminates with supraclavicular lymphadenopathy.
Diagnosis?
A. Teratoma (ans)
B. Sarcoma
C. Hematocele
D. Testicular torsion
E. Seminoma (??)

127. Female recently moved back to UK after living in Australia for 10 years. Has a lymph
node at upper femoral triangle. Diagnosis?
A. Anal cancer
B. Cervical cancer
C. Malignant melanoma (ans)

128. Most commonly incised structure during a Pfannenstiel incision?


A. Rectus sheath (ans)
B. Rectus abdominis
C. Arcuate line
D. External oblique aponeurosis
E. Transverse abdominis muscle (yes muscle not fascia)

129. Boy with bilateral palpable testis with irreducible groin swelling that transilluminates.
Diagnosis?
A. Hydrocoele of cord (?ans)
B. Direct inguinal hernia
C. Indirect inguinal hernia
D. Varicocele

77
130. Origin of deep inguinal ring?
A. Transversalis fascia
B. External oblique aponeurosis
C. Internal oblique muscle

131. 35-year-old has groin swelling he found to have hernia which is lateral and below to the
pubic tubercle. What is the medial boundary of femoral canal?
A. Lacunar ligament (ans)
B. Pectineus
C. Inguinal ligament

132. Structure lateral to direct inguinal hernia?


A. Inferior epigastric vessels

133. What is true regarding femoral canal?


A. Lateral has femoral vein

134. 10-year-old boy with right-sided limp. Severe decrease in all ROM of hip. Was unwell
one week ago. Temperature of 39°C. Diagnosis?
A. Septic arthritis (ans)
B. Transient synovitis
C. Perthes’ disease

135. Seven-year-old boy with knee pain and limited hip movements. Diagnosis?
A. Perthes’ disease
B. SUFE

136. 13-year-old boy with three-month history of right knee pain and limited hip movements.
Right leg is 1.5cm shorter than left leg. Otherwise well. Diagnosis?
A. Perthes’ disease
B. SUFE

137. Seven-year-old boy with knee pain and limited hip movements. Diagnosis?
A. Perthes’ disease
B. SUFE

138. Chyle leak following oesophagectomy and now for re-exploration of chest to ligate the
thoracic duct. Where should clamp be applied to the thoracic duct?
A. Aortic hiatus
B. oesophageal hiatus

139. 10-day old baby who on the third day of life had to have meconium evacuated by
suppositories. Now has bilious vomiting and distended abdomen. Diagnosis?
A. Hirschsprung disease

78
B. Meconium ileus

140. Female with neck mass. Exam reveals a 2cm thyroid nodule and no other symptoms.
FNAC benign. Best management of this case? (did not mention that a US was performed).
A. Hemithyroidectomy (ans, recall q)
B. Discharge
C. Review U/S
D. Total thyroidectomy

141. Female at Endocrine Clinic with a right-sided 2cm thyroid nodule and no lymph nodes on
clinical exam. First investigation you will request for in this case is?
A. Ultrasound
B. FNAC
C. Thyroid antibody
D. Radioisotope scan

142. 45-year-old patient with 2cm thyroid mass and right cervical lymphadenopathy. Biopsy
shows medullary thyroid cancer. Best management?
A. Total thyroidectomy
B. Total thyroidectomy with selective nodal excision (ans)
C. Hemithyroidectomy

143. 62-year-old female with thyroid nodule for three months. Now has metastatic fracture
of femur. No other medical conditions. Diagnosis?
A. Papillary
B. Follicular (ans)
C. Lymphoma
D. Medullary
E. Anaplastic

144. Male patient with bilateral cervical lymphadenopathy. Smoker. Lymph node biopsy
shows increased mitosis and some other malignant features. Primary site?
A. Tongue (ans)
B. Buccal mucosa
C. Palatine tonsil
D. Ethmoid sinus

145. Premature neonate is taken to Special Care Unit with cyanosis. Diagnosis?
A. Transposition of great vessels (ans)
B. Patent foramen ovale

146. Car repair guy went to GP with lump on pulp of his finger. Firm and nontender and has
intact overlying skin. Started after he pricked his hand on a sharp object weeks ago. X-ray
shows no foreign body. Diagnosis?

79
A. Epidermoid cyst (?ans)
B. Glomus tumour
C. Foreign body granuloma
D. Pulp abscess
E. Ganglion

147. Thorn prick injury with vesicular cellulitic rash. Organism?


A. Strep. pyogenes (ans)
B. Bacillus anthracis
C. Klebsiella

148. Patient arrived for elective inguinal hernia repair. Temperature of 35.4°C. Next step?
A. No general anaesthesia until temperature is back up to 36°C (ans)
B. General anaesthesia with forced air warming
C. General anaesthesia with forced air warming and warm fluid
D. Local anaesthesia with local warming devices

149. In a patient post cardiac transplant who wants to join a cardiac rehabilitation program
me. Which factor is known to increase cardiac output?
A. Increased atrial filling (ans)

150. Female with pain and swelling of lateral aspect of knee. History of "giving way". Knee is
flexed at 30° and passive flexion and extension are limited by pain. No history of trauma.
Diagnosis?
A. Lateral meniscal injury
B. LCL
C. Loose body (?ans; no history of trauma)
D. ACL
E. Patellar dislocation

151. Obese male with polyuria and a random blood glucose of 11.9mmol. Diagnosis?
A. Diabetes (ans)
B. Impaired fasting glucose
C. Impaired glucose tolerance

152. Gunshot to lateral side of knee. Structure most likely to be damaged?


A. Common peroneal nerve

153. A patient with Pancoast tumour causing Horner’s syndrome. Horner syndrome. What is
damaged?
A. Posterior root of sympathet
B. Lateral root of sympathetic (ans)
C. Anterior root of sympathetic
D. Edinger Westphal nucleus

80
E. Posterolateral root

154. Structure anterior to right adrenal gland?


A. IVC (ans)
B. Left lobe of liver
C. Stomach
D. Kidney

155. Male after road traffic accident, bilateral femur fracture. Treated and intubated. Day 10
suddenly deteriorates. Petechial hemorrhages. Hb normal. fibrin low, platelet low, PT > 60
seconds:
A. Fat embolism
B. Pulmonary embolism
C. DIC (ans)

156. Patient with four-day history of progressive cough and shortness of breath. Has right
lower lobe dullness. ABG: pH 7.41, pO2 normal, pCO2 low, HCO3 normal. Diagnosis?
A. Pulmonary embolism
B. Pneumococcal pneumonia (ans)
C. Pulmonary oedema
D. Aspiration pneumonia

156. Scenario of patient post RTA. Indication for renal replacement therapy?
A. K > 6.9 (ans)
B. pH < 7.25
C. Urine output 50ml/hr
D. Urea 25 mmol/L

157. Head injury and the patient needs long term feeding (no basal skull fracture). Most
appropriate option?
A. Gastrostomy (ans)
B. Fine NG tube
C. Nasojejunal tube

158. Patient has erectile dysfunction after anterior resection of rectal tumour. Why?
A. Damage to pelvic splanchnic nerves

159. Brachial cyst is derived from which arch?


A. First
B. Second (ans)
C. Third
D. Fourth

160. Inferior parathyroid is derived from which structure?

81
A. Third cleft
B. Second cleft
C. Third pouch (ans)
D. Fourth pouch
E. Third arch

161. 17-year-old patient in ED with severe chest pain. Chest X-ray shows widening of
mediastinum. Where is lesion?
A. Ascending aorta
B. Descending aorta
C. Aortic arch

162. Mother notices brownish colour fluid in front of nappy. Baby is otherwise well.
Diagnosis?
A. Patent vitellointestinal duct (ans)
B. Omphalocele
C. Patent urachus
D. Umbilical hernia

163. Patient with trauma to lower limb. Hip is internally rotated with absent movement
below knee. Diagnosis?
A. Posterior hip dislocation with sciatic nerve (ans)
B. Anterior dislocation with femoral nerve
C. Fractured neck of femur

164. Enzyme involved in autodigestion of pancreas?


A. Trypsin (ans)
B. Amylase
C. Lipase

165. Trendelenberg test positive. Muscle?


A. Gluteus medius
B. Gluteus maximus
C. Gluteus minimus

166. Oropharyngeal carcinoma is associated with which virus?


A. CMV
B. HPV (ans)
C. EBV

167. Level of oesophageal opening in diaphragm?


A. T10 (ans)

168. Patient asked you the important or basis of ERAS?

82
A. Increase Nitrogen balance and ecrease insulin resistance (?ans)

169. Patient with long term Crohn’s disease has abdominal pain and loss of appetite. Albumin
is 29 (low). What feeding?
A. Peripheral TPN
B. Central TPN
C. High protein diet
D. Elemental diet
E. Multivitamin supplement

170. 43/45-year-old female with tender, mobile lump of right inner lower breast. Has erratic
menstrual cycle. Diagnosis?
A. Sclerosing adenosis (ans)
B. Pseudoangiomatous stromal hyperplasia
C. Benign breast cyst
D. Fibroadenoma
E. Ductal carcinoma

171. Biopsy findings in Barrett oesophagus?


A. Non-keratinized stratified squamous epithelium
B. Columnar with goblet cells (ans)
C. Cuboidal epithelium

172. Oesophageal perforation from upper GI endoscopy. 5 days have since passed.
Investigation to check for healing?
A. Barium swallow
B. Water soluble contrast (ans)
C. CT
D. MRI
E. Upper GI endoscopy

173. Patient with diarrhoea and atrial fibrillation of 140 beats/minute. Which electrolyte
imbalance?
A. Hypokalaemia (ans)
B. Hyperkalaemia
C. Hyponatraemia
D. Hypocalcaemia

174. Fit elderly male pain in leg after walking 100m. Finds it easier to walk uphill and cycle.
Diagnosis?
A. Lumbar canal stenosis (ans)
B. Central disc prolapse
C. Lateral disc prolapse

83
175. Male hit in eye by a squash ball. Has diplopia. X-ray and fundoscopy are normal. Next
step?
A. CT of bony structures
B. MRI of bony structures

176. ECG changes in severe hypothermia?


A. J waves between QRS and T waves (ans)

177. Post-operative laparotomy scenario. Urine output 80ml/kg for 24 hours. Dry abdominal
drain with no nasogastric aspiration. Will be NBM for the next day. Which fluid regime?
A. 1L normal saline 0.9% and 1.5L dextrose 5% (ans)
B. 1L 0.9% saline and 1.5L of 0.18% NaCl/0.45% dextrose
C. 1L normal saline 0.9% and 1.5L Hartman
D. 2L 0.18% NaCl/0.45% dextrose

178. Patient with swelling behind the ear, has fever, neck stiffness and convulsions. Has a
history of right ear pain and discharge. What is cause of patient’s acute condition?
A. Cholesteatoma
B. Meningioma
C. Cerebral abscess (ans; complication of cholesteatoma)
D. Intracranial tumor

179. Patient with perforated bowel resuscitated with 20ml/kg fluids. SVR low, CO high, CVP is
8 (normal 2-8). Type of shock?
A. Anaphylactic
B. Septic (?ans; low SVR in infected patient points to septic shock)
C. Hypovolemic
D. Cardiogenic
E. Neurogenic

180. Location for femoral artery aspiration for sample?


A. Asis and pubic symphysis
B. Asis and pubic tubercle

181. Blood supply to transverse colon?


A. Middle colic

182. Gastric acid secretion is due to a combination of?


A. Gastrin, vagal, histamine (ans)
B. Gastrin, vagal, secretin
C. Gastrin, CCK, secretin

183. Endoscopy shows blood from the first part of the duodenum. There is also a deep ulcer
posteriorly. Which blood vessel is responsible for the bleeding?

84
A. Gastroduodenal (ans)
B. Superior pancreaticoduodenal
C. Inferior pancreaticoduodenal
D. Right gastric

184. Young male very anxious about his upcoming operation. Tachycardic. Physiology?
A. Adrenaline released from medulla adrenal (ans)
B. Noradrenaline released from medulla adrenal
C. Adrenaline released from sympathetic system
D. Noradrenaline released from sympathetic system
E. Increased amount of circulating noradrenaline

185. Sigmoid surgery. Best method for reducing risk of post-op infection?
A. Immediate pre-op shaving
B. Pre-op skin cleaning with antiseptic solution
C. Ultra clean air
D. Zoning of theatre
E. Bowel preparation

186. 5-year-old patient brought by his mother who noticed he developed scrotal swelling
after playing football. What is responsible?
A. Congenital hydrocele
B. Inguinoscrotal hernia
C. Inguinal hernia
D. Patent process vaginalis
E. Failure of gubernaculum

187. Annular pancreas affects which part of duodenum?


A. First part of duodenum
B. Second part of duodenum (ans)

188. Patient post RTA. CVP 15, BP 170/110, PCWP 18. No urine in catheter in last six hours
from admission. She is getting increasingly agitated and hypertensive and requires increasing
sedation. Most likely cause?
A. Blocked catheter (ans)
B. Hypovolemia
C. Bilateral ureteric injuries/hydronephrosis
D. Renal failure
E. Heart failure

189 .Patient with lung cancer has persistent headache from brain metastasis. Treatment?
A. Corticosteroids (?ans; as headache can be due to oedema around mets)
B. Paracetamol
C. NSAIDs

85
D. Slow-release morphine
E. Radiotherapy

190. Name for volume from maximum inspiration to maximum expiration?


A. Total lung capacity
B. Vital capacity (ans)

191. Spirometry changes in patient with COPD?


A. Increased residual volume

192. Eight-year-old child with groin pain. Both hips have limited motion at extreme ends of
hip range of motion. Diagnosis?
A. Missed DDH (ans)
B. Perthes’ disease
C. Transient synovitis

193. One-day-old male baby with bilious vomiting and scaphoid abdomen. Vomiting starts
after feeding. No mass felt on palpation. Diagnosis?
A. Duodenal atresia (ans)
B. Pyloric stenosis
C. Intussusception

194. Female with two year history of intermittent right upper abdomen pain. Now has right
upper abdomen pain and fever. LFT and CBC normal. Next step?
A. ERCP
B. MRCP
C. Ultrasound
D. Abdominal X-ray
E. CT scan

195. Seven-year-old girl brought for a tonsillectomy. Otherwise well. Which of these are
required preoperatively?
A. CBC
B. Renal function test
C. Coagulation profile
D. All of the above
E. None of the above (ans)

196. Elderly man with bypass grafting done. Used both internal mammary arteries. Now has
pain in his arm while hanging clothes. Why?
A. Stenosis of subclavian proximal to 1st branch (ans)
B. Stenosis of subclavian distal to 1st branch

197 .Patient with dysphagia. Diagnosed with gastric lymphoma. What cell type?

86
A. B cell
B. T cell
C. NK cell
D. Macrophage

198. Patient involved in RTA. Has cerebral oedema. 16 hours increased urine output to
350ml/hr. Urine osmolality high, plasma osmolality low, hyponatremia. Which part of
nephron affected?
A. PCT
B. DCT
C. Descending limb
D. Ascending limb
E. Collecting duct (ans)

199. Patient in RTA. On applying painful stimulus, he opens eyes, groans and has
incomprehensible speech. What is the GCS?
A. 9 (ans; E2V2M5)
B.11

200. Infection from a mastoid abscess can spread to which of these?


A. Internal carotid artery
B. Inner ear
C. Occipital sinus
D. Sigmoid sinus (ans)
E. Facial nerve

201.Clamping of the free edge of lesser omentum. Structure directly injured is?
A. CBD (ans)
B. Cystic artery
C. Hepatic vein

202.35-year-old woman presents with recurrent peptic ulceration. She is on proton pump
inhibitors and previously received Helicobacter pylori eradication therapy three months ago.
Which of the following is likely to be raised on venous blood testing?
A Cholecystokinin
B Gastrin
C Histamine
D Pancreozymin
E Secretin

203. A 55-year-old man presents with back ache. Neurological examination reveals lack of
extension

204. Structure at most risk of injury during ligation of splenic hilum?

87
A. Tail of the pancreas (ans)
B. Greater curvature of stomach

205. Structure liable for direct injury while ligating short gastric vessels in splenectomy?
A. Fundus stomach
B. Tail of pancreas (ans)
C. Left diaphragm
D. Left kidney
E. Left adrenal

206. Relation of ulnar artery to ulnar nerve?


A. Radial to ulnar

207. Patient has a hoarse voice post endarterectomy. Which nerve is injured?
A. Vagus
B. Hypoglossal

208. Postero-lateral approach to the ankle. Which nerve is injured?


A. Sural
B. Saphenous

209. Post gastric bypass. Patient has dizziness and palpitations. Cause?
A. Hypoglycaemia

210. Mechanism of bradycardia in Cushing triad?


A. Increased activity from carotid body
B. Decreased activity from carotid sinus
C. Decreased activity from cardiopulmonary receptors
D. Increased activity from aortic sinus

211. Patient involved in head trauma. What will reflect his problem (Chart given)
A. BP increased, HR decrease

212. Patient with head injury. Hypertensive and bradycardic. What are the changes in
baroreceptor firing and parasympathetic activity to cause the bradycardia?
A. Decrease, decrease
B. Increase, increase
C. Decrease, increase
D. Increase, decrease

213. During surgery for Dupuytren contracture of ring finger, a nerve is injured lateral to long
flexor tendons at level of MCP joints. Nerve affected?
A. Dorsal branch of ulnar
B. Digital branch of median

88
C. Recurrent branch of median
D. Deep branch of ulnar
E. E. Superficial branch of ulnar

214.Site of action of morphine?


A. Mu receptors

215.Glass cutting proximal to distal palmar crease. Loss of proximal joint flexion of thumb.
What is associated with it?
A. Loss of sensation over palmar thumb and two and half fingers
B. Loss of sensation over the whole palmar

216. 35-year-old female with antithrombin 3 deficiency develops DVT 10 days post abdominal
surgery. Management?
A. Heparin
B. Warfarin for life (?ans)
C. Warfarin for six months

217. Athlete with pain on deep flexion. Dx labral tear. Investigation?


A. MR arthrogram
B. CT
C. X-ray
D. USS

218. Where best to do an emergency tracheostomy?


A. Incise cricothyroid membrane
B. Traction on cricothyroid muscle

219. Female with spasms and tetany. Most likely electrolyte abnormality?
A. Hypocalcemia
B. Hyponatremia

220. Patient underwent anterior resection. Post op has pain, spasms and numbness on medial
side of thigh. Nerve affected?
A. Ilioinguinal nerve
B. Obturator nerve
C. Femoral nerve

221. Eight-month-old boy. Where does majority of blood from IVC go to after the right
atrium?
A. Left atrium
B. Left ventricle
C. Right ventricle

89
222. Pathology of AAA?
A. Atherosclerosis

223. Elderly lady with back pain for many months. Lost 2cm height over past ?two years.
Corrected Ca high, phosphate upper end of normal, PTH high, eGFR low. Diagnosis?
A. Primary hyperparathyroidism
B. Secondary hyperparathyroidism
C. Tertiary hyperparathyroidism (ans)
D. Psuedohyperparathyroidism

224. What helps surgeon identify uncinate process of pancreas?


A. Lies anterior to IMA
B. Lies anterior of SMV
C. Lies posterior to SMA
D. Lies anterior to aorta

225. Nerve damaged during surgery to remove submandibular duct stone?


A. Lingual (ans)
B. Facial

226. Loss of sensation to one side of the ant 2/3 of tongue (Post Op). What nerve is injured?
A. Lingual
B. Hypoglossal
C. Facial
D. Vagus
E. Glossopharyngeal

227. By what mechanism of action does ibuprofen cause ulcers?


A. Prostaglandin inhibition

228. Heparin mechanism of action?


A. Thrombin inhibition

229. ECG findings in PE?


A. ST depression
B. T wave inversion in V1-V3
C. LBBB
D. Left axis deviation

230. Hypocalcaemia ECG findings: Prolonged QT interval

231. Cardiac valve composition?


A. Dense fibrous tissue
B. Haylinevcartilage

90
232. Anterior compartment syndrome due to tibial fracture. Where will he develop sensory
problem
A. First web space
B. Dorsum of foot

233. Dilated pupil due to?


A. Unopposed sympathetic supply to dilator pupillae

234. Child with painless rectal bleeding. Diagnosis?


A. Juvenile polyp

235. ABG showing respiratory acidosis.

236. ABG showing compensated metabolic alkalosis.

237. ABG showing metabolic alkalosis

238. Splenic trauma. Chest X-ray

Diagnosis?
A. ARDS (ans)

239. What structure is in contact with T5 vertebra?


A. Azygos vein
B. Trachea bifurcation
C. Ascending aorta
D. Descending aorta

240. Child with blue cystic swelling in floor of mouth. Diagnosis?


91
A. Ranula

241. Structure related to cuboid?


A. 4th and 5th metatarsals

242. Gout
A. Negative birefringent needle crystals

243. First MTP pain with osteophytes and subchondral cyst. Diagnosis?
A. Osteoarthritis (ans)

244. What constricts efferent arteriole?


A. Angiotensin II (ans)

245. Inferior mesenteric vein drains into?


A. Splenic vein

246. Another 4 rib fracture with low pO2, normal pCO2. What next?
A. CPAP
B. SIMV
C. Analgesia

247. Fawzia recall on decreased renal perfusion

248. What is most common about prostate cancer?


A. 70% have metastasis on diagnosis
B. Metastasises via lymphatics
C. Bone metastasis (ans)

249. Patient on heparin now has bleeding. Why?


A. Thrombocytopenia
B. Factor X deficiency

250. Male with constipation and pain. Blood on tissue. Diagnosis?


A. Anal fissure

251. Child with constipation and pain. Diagnosis?


A. Anal fissure

252. What maintains vascular tone?


A. Renin
B. Angiotensin II
C. Angiotensinogen

92
253. Patient with ulcerative colitis develops an ulcer next to stoma. Diagnosis?
A. Pyoderma gangrenosum (ans)
B. Erythema nodosum

254. Patient 18-months post renal transplant with infection. Organism?


A. EBV
B. CMV
C. HTLV

255. What is true about the spleen?


A. Haemopoiesis in normal individual
B. Contains macrophage
C. Primary lymphoid organ
D. Contains medullary sinuses

256. Origin of internal spermatic fascia?


A. Transversalis fascia

257.Patient with osteoporosis. Electrolyte findings?


A. Normocalcaemia (ans)

258.Elderly patient with history of syncope and ejection systolic murmur. Diagnosis?
A. Aortic valve calcification
B. Mitral valve stenosis
C.

259.Post traumatic median nerve injury at the wristebv. Muscle affected?


A. Abductor pollicis brevis

260.Least reliable test for hypoperfusion?


A. Pulse oximeter (ans)

261.Patient going for surgery is anxious. What is the effect of adrenaline?


A. Bronchodilation
B. Miosis

262.Systemic effect of stress?


A. Increase protein break down

263.A 37-year-old man presents with severe headache, photophobia arid neck stiffness. A
lumbar puncture is performed immediately to determine if a cerebrospinal fluid (CSF)
pathogen is involved. In performing this procedure, which is the first of the following
structures to be pierced by the lumbar puncture needle?
A. Anterior longitudinal ligament

93
B. Arachnoid mater
C. Dura mater
D Interspinous ligament
E. Posterior longitudinal ligament

264. A patient fell on the outstretched hand developed median nerve injury at the wrist.
What is the likely cause?
A. Lunate dislocation
B. Scaphoid fracture
C. Hamate dislocation

265.A patient received a Cadaveric graft. What type of graft is it?


A. Allograft
B. Autograft
C. Isograft
D. Xenograft

266. Dartos muscle location


A. Subcutaenous tissue
B. Scrotal skin
C. Deep to Tunica vaginalis

267.Salivary gland tumor with perineural invasion


A. Adenoid cystic
B. Warthin’s tumour
C. Pleomorphic Adenoma

268.Pigmented lump 1cm in size at the back suspected to be Malignant melanoma.


A. Excision biopsy 1mm
B. Excision biopsy 2mm
C. 1cm excision biopsy

269. Case of diffuse breast enlargement tender and not responsive to antibiotics (
inflammatory carcinomatosis was not in the options )
A. Phylloides
B. Ductal Ca
C. Inavsive Lobular

270. Vomiting electrolyte derangement


A. Hypokalemic Hyponatremic Hypochloremic metabolic alkalosis

271. Burn fluid therapy guided by


A. CVP
B. Lactate

94
C. Urine Sodium

272. Right ventricle relation


A. Diaphragm

273. Hypercalcemia management?


A. Saline infusion.

274. Appendicitis predominant cells?


A. Neutrophils
B. Macrophage
C. Lymphocytes

275. Cannot flex distal phallanx, what tendon is implicated?


A. FDP
B. Lumbricals

276. Anaphylaxis predominant cells found is?


A. Mast cells
B. Eosinophils
C. Basophils

277. Popliteal artery stenosis followed by acute limb ischemia, what is responsible ?
A. Thrombosis
B. Emboli
C. Atheroma
D. Vasculitis

278.Pathophysiology of ARDS :
A. Diffusion defect

279. Burn patient with hiccups and abdominal distension. What is responsible?
A. Acute gastric dilatation.

280.Patient sustained RTA with stable pelvis fracture and head trauma, what will increase
next day?
A. RBC count
B . CRP
C albumin
D.TroponinT

281. MCL injury scenario

95
282.Injury on the level of lateral malleolus- the surgical intervention made by incising
between the tendo-achilles & lateral maleolus what the structure could be injured.
A. Sural nerve
B. Superficial peroneal nerve
C. Saphenous nerve

283. A patient has pain months or years of Pinning done for femoral neck fracture. What is
responsible?
A. Avascular necrosis
B. Non union
C. Malunion

284. 6/12 old child with abdominal pain and rectal bleed, no mass palpable. Daignosis?
A. Midgut volvulus
B. Intussuception

285. A patient stabbed in the chest, Pt is tachycardic but BP is normal and CXr is not
significant. Next step of action?
A. Echo.
B. CT.
C. Immediate transfer to theatre

286. A diabetic patient had an above knee amputation, and now develops hypotension,
tachycardia, tachypnea. Wound is edematous. What is responsible?
A. Gas gangrene
B. Pulmonary embolism
C. Wound infection

287. A patient had a superficial skin cut, the surrounding wound became stiff and indurated.
(The wound later suppurate I think…)What is likely?
A. Gas gangrene
B. Strep Pyogenes

288. 18-year-old female with abdominal pain, weakness and vomiting. Amylase 110. Other
bloods are normal. Diagnosis?
A. Acute intermittent porphyria

289. What is the first response in low arterial pressure?


A. Baroreceptor
B. Chemoreceptor

290. A question about paraneoplastic syndrome in a patient with lung mass.What will be
present in serum?
A. Parathyroid hormone-related peptide (PTHrP)

96
B. Parathyroid hormones

291. Primary hyperparathyroidism table. The one that came in mock.

292. 2cm adrenal tumor in imaging: Diagnosis?


A. Cortical non-functional adenoma
B. Functioning adenoma
C. Phaechromocytoma

293. Dehydrated patient. What do u expect in Urine?


A. Increased Sodium
B. Specific gravity of 1001 (not decimals)
C. Dilute urine

294. Weakness of muscles of mastication?


A. Foramen ovale

295. There was a question with answer as facet arthropathy

97

You might also like